You are on page 1of 21

REFERENCE: NELSON, TEXTBOOK OF PEDIATRICS, 17 TH EDITION 13.

13. A 6 year old male was seen in the ER due to Tea colored urine of 2 days duration. This
was associated with periorbital edema, abdominal distention and grade 2 pitting edema
Case I. Nos. 1-2 of the lower extremities.
Impression : Acute Glomerulonephritis/PSGN
A 3 year old normal child was admitted to a hospital because of simple febrile convulsions Post streptococcal glomerulonephritis is most common in children aged: P 1740
A. 5 - 10years old
1. Routine management of the case includes: B. 2 - 4 years old
C. 13-18 years old
A. careful search for cause of fever* (0.5)
D. 1-2 years old
B. short term anti-convulsant prophylaxis (0.5)
C. Phenobarbital maintenance to prevent recurrence 14. The best single antibody titer to document cutaneous streptococcal infection in PSGN
D. Lumbar puncture is:
2. The anti-convulsant that may be effective in the acute management of prolonged febrile seizures A. Deoxyribonuclease B antigen (DNase)
is: B. ASO titer
A. rectal Diazepam* C. Streptozyme test
B. rectal Phenobarbital D. Phadebact test
C. Intravenous Phenobarbital
D. Intravenous Phenytoin A term neonate with an uncomplicated birth history was noted to be cyanotic during the first few days of
life. On examination, there was central cyanosis and absence of heart murmur. Chest x-ray
Case II. Nos. 3-4 showed normal heart size and diminished pulmonary vascular markings. Most likely diagnosis is
A two-week old baby was noted to have a rapidly increasing head circumference. At birth, head A. Pulmonary Valve Atresia
circumference was 40 cms, against a chest circumference of 34 cms. Present physical examination B. Tetralogy of Fallot
showed a head circumference of 45 cms, wide, bulging anterior fontanel, gaping sutures, dilated scalp C. Truncus Arteriosus, type I
veins and a (+) setting sun sign. D. Transposition of Great Arteries with VSD
3. If the occiput is prominent, the primary consideration is: p 1489 A 3 month old boy was noted to be cyanotic on crying when he was 2 months old. This became
A. Aqueductal stenosis persistent and more pronounced on exertion. On physical examination, the baby was cyanotic with
B. Chiari malformation a grade 2 3/6 systolic ejection murmur at the 3rd 4th ICSLPSB. This baby is suffering from
C. Dandy-Walker malformation*
A. Pulmonary Valve Atresia
B. Tetralogy of Fallot
D. Hydranencephaly C. Transposition of Great Arteries, no VSD
4. This drug will reduce the rate of CSF production: p1490 D. Tricuspid Valve Atresia
A. Dexamethasone
B. Acetazolamide*
C. Prednisone A 4month old baby girl was brought for consultation because of frequent cough and
D. Mannitol colds accompanied by inability to consume her milk formula, fast breathing during feeding and
chest retractions. A murmur was heard accompanied by bounding peripheral pulses and wide
Case III. Nos. 5 9 pulse pressure. The most likely diagnosis is p 1511
A. Atrial Septal Defect
A 9 year old child was brought to the OPD clinic because of deterioration in school preformance, frank B. Ventricular Septal Defect
dementia, myoclonic jerks and cerebellar ataxia. He had no immunization and contracted measles at 10 C. Patent Ductus Arteriosus
months of age, varicella at 2 years and mumps at 5 years D. Pulmonic Stenosis

5. The most likely diagnosis is: A 2 year old boy presented with a history of 5 days fever accompanied by irritability, bilateral conjunctival
A. progressive rubella panencephalitits injection, unilateral cervical lymphadenopathy, rashes in the diaper area and congested buccal
B. subacute sclerosing panencephalitis mucosa. In order to prevent complication, this boy should be given high dose aspirin and
C. post-varicella encephalitis
D. adrenoleukodystrophy A. Digoxin
6. The seizures are best controlled by: B. Penicillin
A. Carbamazepine C. Diuretics
B. Valproate D. Intravenous immunoglobulin
C. Phenytoin
D. Lamotrigine A 6 year old child was diagnosed as a case of acute rheumatic fever and received ten days course of
aqueous penicillin. In order to prevent the recurrence of group A streptococcal infection, this child
Administration of this drug may prolong the childs survival: p 844 should receive
A. Methisoprinol A. Benzathine benzylpenicillin 1.2 million units intramuscular once
B. Inosiplex B. Benzathine benzylpenicillin 1.2 million units intramuscular every 21-28 days (1.0)
C. Taurine C. Oral penicillin 250 mg twice a day for 10 days
D. Intravenous immunoglobulin D. Oral erythromycin 250 mg twice a day for 10 days

The most important hormone regulating renal calcium excretion is: p 200 A 10 year old boy who presents with difficulty in breathing accompanied by a displaced PMI to the left,
A. Dihydroxy vitamin D apical and systolic thrill and grade 4/6 holosystolic murmur at the apex, is most likely suffering from
B. Calcitonin an insufficient p 1570
C. Parathyroid hormone A. Aortic valve
D. Thyrotoxin B. Mitral valve
C. Pulmonic valve
A 2 year old female was noted to be oliguric for 24 hours. She was brought to the ER with Creatinine D. Tricuspid valve
levels of 350 mmol/l. Serum potassium level was 6.5 mmol/l. What possible maneuver can induce
a negative potassium balance and decrease serum potassium levels? Children with small Ventricular Septal Defect are at risk for this complication
A. Hypoxic spells
A. sodium bicarbonate infusion B. Heart failure
B. insulin/glucose infusion C. Pulmonary Artery Hypertension
C. calcium gluconate D. Infective Endocarditis
D. kayexalate

10. A 1 year old baby boy was brought to the ER due to seizure. On PE, the baby was Case 1 Nos. 22 24
seem to have flexed wrist, fingers extended, thumbs adducted over the palms and the feet
extended and adducted. What is the possible cause of the seizures? P 224 Lito, a 5 year old child, was seeking enrollment in a nursery school. Both the teacher and
A. grand mal seizure the school physician evaluated him for growth and development. Birth history revealed that Lito was born
B. meningitis prematurely at 34 weeks with a weight of 1924 grams and a length of 42 cms. Head circumference was
C. benign febrile seizures 31 cms. The expected anthropometric measurements includes:
D. hypocalcemia
22. Weight that has increased at least _________ from birth
11.A 3 year old boy has been having diarrhea for 5 days already with stooling of 3 5 x per day, A. 4x
voluminous, watery in character. On PE, he was noted to be in moderate dehydration, with serum B. 6x
sodium lever of 160 mmol/l. Patient was hydrated with D5water, however, after 2 hours of C. 8x
hydration. The patient went into a seizure. What is the possible cause of the seizure? D. 10x
23. An increased in length of
A. Patient developed subdural effusion due to the hyperosmolality A. 22 inches
B. There is an excess movement of water into cerebral cells during rehydration causing B. 24 inches
cerebral edema C. 26 inches
C. The patient had late manifestation of hypernatremia D. 28 inches
D. Patient had meningitis 24. Ideal head circumference for age is
A. 40 cms
12. A 2 year old male was seen in the ER due to fever of 5 days duration. This was B. 42 cms
associated with vomiting and anorexia. There were no cough, colds associated. CBC C. 44 cms
revealed leukocytosis with predominance of Segmenters. Urinalysis revealed TNTC pus D. 46 cms
cells and 5-10/hpf RBC. Impression : UTI. The gold standard for the diagnosis of UTI is:
Case 2. Nos. 25 - 29
A. Urine Culture and sensitivity
B. Leukocyte esterase test Buboy is a 7 month old infant brought tot the clinic for a well baby visit. His grandmother
C. Urinalysis recalled that his birthweight was 6 lbs but was not aware of the birth length and head circumference.
D. Nitrite test
25. Buboys ideal length is
A. 24 inches
B. 27 inches C. 600 800 ml
C. 31 inches
D. 35 inches D. 800 1000 ml
26.The expected increase in his head circumference is:
A. 2 cms Case 1. No. 40
B. 4 cms
C. 6 cms A 2 year old child was brought to your clinic with a history of having fallen down the stairs from a height of
D. 8 cms 6 steps. You would suspect child abuse because of the presence of the following:
27.Buboys ideal weight at 6 months is
A. 11 lbs 40. A. bruises on the back
B. 13 lbs B. 2 cm skull fracture parietal area
C. 15 lbs C. clavicular fracture medial aspect
D. 17 lbs D. contusion/abrasions on the upper arm and legs
28.When Buboys is offered an object he is expected to
A. use his index finger to get it REFERENCE: APMC Workbook on Child Protection
B. get the object and drop it
C. cast the object A 2 year old girl was brought to the ER with a history of paracetamol ingestion. Mother claimed that 3
D. transfer the object form hand to hand hours PTC, the child ingested an almost full 30 ml bottle of Paracetamol with a concentration of 250
29.The motor milestone expected for age is mg./kg. the child weighed 10 kg.
A. sitting propped up on hands
B. crawling 41. This statement is TRUE
C. Pulling up to stand A. She has ingested a toxic dose of paracetamol
D. Cruising B. Children have a lower incidence of toxic plasma levels after ingestion than do
adolescents
30. A 9 year old boy is suffering from greasy foul watery stool of 2 weeks duration accompanied C. Ipecac should be given as an initial GI decontaminant
with abdominal cramps and abdominal distention. You are entertaining the possibility of Giardiasis D. Activated charcoal should not be given if NAC is to be administered
but the stool examinations done thrice were negative. The next best procedure to do is :
42. Initial symptom include:
A. abdominal ultrasound P 2366 - 2367
B. entero test or string test A. oliguria
C. duodenal biopsy B. jaundice
D. polymerase chain reaction C. diaphoresis
D. abdominal pain
31. A 2 year old girl was brought in for consultation because of watery stool and
vomiting of 3 days duration. Modified acid fast staining of the stool revealed a 2- A 10 year old child, diagnosed to have Juvenile Rheumatoid Arthritis and maintained on salicylate was
6 micrometer red oocysts . The most likely diagnosis in this case is : brought to the ER with fever, headache and body malaise. You considered the possibility of salicylate
A. giardiasis toxicity
B. balantidiasis
C. cryptosporidiosis 43. This statement is TRUE:
D. amebiasis A. Serum salicylate levels should be taken 6 hours after the last salicylate ingestion
B. Chronic toxicity usually presents with metabolic acidosis
32. A 4 year old girl with protuberant abdomen has recurrent colicky periumbilical (1.0)
pain. Fecalysis shows a broadly ovoidal ova with thick shell and outer C. A salicylate level of 10 20 mg/dl will rule out chronic toxicity
mammilated covering about 40-60 micrometer. Because of high rate of D. Gastric decontamination should be performed immediately
reinfection, chemotheraphy has to be repeated at : 44. The management of salicylate toxicity would include the administration of
A. 1 month interval A. Potassium
B. 2 months interval B. Calcium
C. 3-6 months interval C. Acetazolamide
D. 8-12 months interval D. Mannitol
E.
33.An 18 year old male from Samar presented with abdominal distention , Nos. 45 49: REFERENCE: Nelson, Textbook of Pediatrics, 16th edition
hepatomegaly nad some signs of portal hypertension. Fecalysis revealed small egg
with short curved spine. The specific drug for this condition is : p 1167 45. A major criteria in the diagnosis of atopic dermatitis is
A. diethylcarbamazine P 682
B. albendazole A. chronic relapsing course
C. ivermectin B. elevated serum IgE
D. praziquantel C. early age of onset
D. white dermatographism
34. A 6 month old infant with feeding problem was noted to have diffuse papular lesions , chorioretinitis on
the right eye and enlarged head circumference . IGM-SAGA test was positive. This patient can 46. A 5 year old with asthma who have daytime symptoms of >2 times a week, nocturnal symptoms
best be treated with . p 1164 - 1166 of > 2 times monthly. PEFR > or equal to 809. PEFR variability 20 30% is diagnosed to have
A. pyrimethamine + sulfadiazine + calcium leukoverin
B. Metronidazole A. intermittent asthma
C. Iodoquinol B. mild persistent asthma
D. Chloroquine phosphate C. moderate persistent asthma
D. severe persistent asthma
35. 3 year old boy who owns a puppy has a history of pica. He develops fever, cough with wheezing
and hepatomegaly. The test that that will most likely helps in making an accurate diagnosis is : 47. For controller/maintenance treatment, this patient may need EXCEPT
A. inhaled corticosteroid
B. cromolyn
A. stool examination C. nedocromil
B. ELISA D. oral steroids
C. COPT
D. PCR 48. A child who suddenly develops difficulty of breathing, urticarial rashes, abdominal pain 5 minutes after
eating crabs need immediate administration of
36. A 24 month old infant with a normal weight came in for diarrhea with severe dehydration. You A. intramuscular antihistamine
opted to administer plain LRS. The total amount of fluid that should be administered in 3 hours is B. intravenous antihistamine
approximately C. intramuscular corticosteroid
A. 800 ml D. epinephrine 1:1000 intramuscular
B. 1000 ml
49. The only effective treatment of B-cell disorder
C. 1200 ml A. Thymic cell transplant
B. Bone marrow transplant
D. 1400 ml C. Regular administration of gammaglobulin (0.5)
D. Plasma transfusion
37. A 12 month old infant came in for bloody stool, high fever, severe abdominal pain and toxicity for the
past 32 hours. Stool exam revealed 60 fecal leukocytes, plenty of bacteria and amoebic cysts. REFERENCE: Nelson, Textbook of Pediatrics, 17th edition
Most likely the cause of the diarrhea is
A. Shigella Case 1. Nos. 50 - 51
B. Amoeba (Entamoeba histolytica) Baby girl OA was born to a 24 year old primigravid at 39 weeks AOG, NSD after an
C. Salmonella uncomplicated pregnancy. She weighed 2850 grams, AS 9 & 10, On the 12 th hour of life, she was noted
D. Enteroinvasive Escherichiae coli to be jittery. There was fair suck. Reflexes were good. Serum calcium was 4 mg/dl. HGT was 70 mg%

38. A 3 week old infant came in for non-bilous vomiting occurring immediately after feeding. This was 50. The most likely diagnosis is
followed by progressive loss of fluids and electrolytes and dehydration. Feeding was alright. Most A. Transient idiopathic neonatal hypocalcemia
likely you are dealing with B. Early neonatal hypocalcemia
A. Duodenal atresia C. Small for age
B. Esophageal atresia D. Infant of diabetic mother, hypocalcemia secondary
C. Hypertrophic pyloric stenosis 51. The most probable cause of her disorder is
D. Duodenal stenosis A. metabolic imbalance of hypoglycemia (0.25)
B. presence of cytoplasmic antibodies
39. A fairly nourished one year old in some dehydration will be needing approximately C. functional immaturity of PTG
this amount of rehydration fluid during the initial replacement therapy D. prematurity
A. 200 400 ml
Case 2. Nos. 52 - 53
B. 400 600 ml
A 3 month old male was admitted due to frequent vomiting since 2 weeks of age. He had poor suck,
weighed 3.5 kgs, dehydrated, hypotensive
52. Differential diagnoses includes all of the following EXCEPT C. iron has no significant effect on the hosts defense mechanism (0.25)
A. septic shock D. the high levels of transferring is still enough to facilitate absorption (0.25)
B. adrenocortical insufficiency, etiology to be determined
C. failure to thrive
D. Acute gastroenteritis 66. A newborn was delivered to a mother who is positive for the hepatitis B virus. The following is/are
TRUE regarding breastfeeding of the mother in this infant p 159
53. Management should include A. may breastfeed only once maternal infection is fully cured (0.5)
A. DAT, Prednisone, NaCl supplementation B. should stop breastfeeding because hepatitis B is a highly contagious disease
B. NPO, D5NSS, hydrocortisone succinate, fludrocortisone C. active and passive immunizations permit breastfeeding with little risk to the infant
C. NPO, D5NSS, hydrocortisone succinate D. breast feeding is the major route of transfer of infection to the body
D. NPO, D5NSS, fludrocortisone and NaCl supplementation
67. An eleven (11) year old boy will most likely
Case 3. No. 54 A. groom in favor of the peer group uniform
A 9 year old male had a weigh of 120 lbs, height of 4 feet complains of paroxysmal B. be able to engage in intimate and empathetic relationship with another person
attacks of headache and nape pain. The dark pigmentation on the neck, axilla, nape and his acne C. be idealistic
bothers him a lot. On consultation, his BP was 120/80 mmHg, CR: 90/min D. none of the above

54. The patient has 68. An adolescent boy was seen at the clinic for a routine check up. On history and physical examination,
A. normal weight, height and BP he was noted to be very self-conscious and his peers were all boys. Genital examination showed
B. increased weight, normal height and increased BP scanty long, slightly pigmented pubic hair at the base of the penis and scrotum was slightly enlarged.
C. normal weight, increased height and increased BP Based on the above history and PE one of the following is also expected
D. increased weight, increased height and increased BP
A. voice change is already completed
Case 1. Nos. 55 56 B. presence of hair over the chest
C. peak of penile growth
A diabetic 33 year old Gravida 1 Para 1 type O+ mother delivered 5 weeks earlier than her D. ejaculation usually in response to masturbation
expected date of confinement. The baby was delivered in a lying-in and was noted to be limp, cyanotic
face and pale body, gasping, pulses weak and no response on suctioning 69. In the HEADS FIRST acronym for psychosocial history of adolescence, R stands for
A. Rebelliousness
55. If your are present on the delivery of this baby. The Apgar score at 1 minute is B. Recreation
A. 0 C. Responsibilities
B. 1 D. All of the above
C. 2
D. 3 70. The first pubertal sign in boys is
56. If he was fed on expressed breast milk and no vitamin K given at birth, the anticipatory p 54 - 56
condition that could probably set in A. appearance of mustache
A. Hemorrhage B. broadening of the shoulder
B. Jaundice C. increase muscle mass
C. Diarrhea D. testicular enlargement
D. Necrotizing enterocolitis
71. A 6 year old female was seen at the ER because of fever of 4 days. This was associated with sore
REFERENCE: Routine newborn care throat, decrease in appetite and slight dysphagia. Physical examination revealed hyperemic throat
with petechiae on the soft palate. Complete blood count revealed WBC 14,000 with Segmenters of
REFERENCE: Nelson, Textbook of Pediatrics, 17th edition, 0.78. Your most likely diagnosis is p 1393
A. Bacterial pharyngitis
57. A 20 year old primigravid mother now on her 31st week age of gestation consulted for painless vaginal B. Acute epiglottitis
bleeding. Stat pelvic ultrasound showed placenta previa totalis and thus emergency caesarean C. Right peritonsillar abscess
section was done. What immediate problem do you expect in the baby? D. Acute laryngitis
A. Meconium Aspiration Syndrome
B. Respiratory Distress Type I 72. The drug of choice for the above case is
C. Apnea of Prematurity A. Cloxacillin sodium
D. Respiratory Distress Type II B. Penicillin
58. The pulmonary problems brought about by Respiratory Distress Type I are due to C. Ampicillin
A. low compliance D. Cephalexin
B. high resistance
C. high functional residual capacity 73. A 9 month old male infant was brought to the ER because of DOB of 1 day. He had intermittent low
D. low dead space grade fever for 4 days with nasal discharge and non-productive cough. A day prior to consult, he had
fast breathing and decreased appetite. But he remained to be playful. Physical examination revealed
59. A post term was delivered thru caesarean section because of non-reassuring fetal heart rate RR of 60s/min, T: 38oC, intercostal and occasional wheezing with inferior displacement of a normal-
status. Baby was born through thickly meconium stained amniotic fluid, weighed 2400 grams and sized liver. The most likely etiology of the disease is
had poor apgar score. He is at risk for the following problem: A. Parainfluenzae
A. Respiratory Distress Type II B. Adenovirus
B. Persistent pulmonary hypertension C. Influenzae
C. Meconium plug D. RSV
D. Meconium ileus
74. A 4 month old male infant was noted to have stridor during sleeping for the past 1 month. The stridor
60. One of the following is responsible for decrease pulmonary venous return at birth disappears when the infant is carried in an upright position. There was no associated fever and
A. increase paO2 patient was apparently well since birth. The most likely diagnosis is
B. increase paCO2
C. decrease pH A. Laryngitis
D. all of the above B. Laryngeal foreign body
C. Laryngomalacia
REFERENCE: Nelson, Textbook of Pediatrics, 17 th edition, p 584 D. Bronchomalacia
75. Signs and symptoms of above condition usually resolve by
61. Unconjugated hyperbilirubinemia in an otherwise normal 8 day old infant can be attributed to p1409
A. breastmilk jaundice A. 6 months
B. breastfeeding jaundice B. 12 months
C. sepsis neonatorum C. 18 months
D. galactosemia D. 24 months

62. A 3 year old child with malnutrition is being evaluated for admission at the ward. The 76. A 3 year old healthy child was left alone in the room playing. When the mother went back after a few
anthropometric index that reflects recent nutritional experience of the child is p170 minutes, she saw her child lying on the floor, awake, aphonic and apneic. She was rushed to the ER.
A. weight for height Your most likely diagnosis is complete upper airway obstruction due to foreign body aspiration. The
B. body mass index maneuver/s to do on the way to the hospital is/are P 1411
C. mid-arm muscle circumference A. Heimlich
D. triceps skin fold B. back blows and chest thrusts (0.33)
C. blind finger sweep of the oral cavity (0.33)
63. A 1 year old male consulted the OPD for persistent diarrhea of 2 weeks. Physical D. A and C
examination showed he had dry, scaly eczematous dermatosis at the perioral, cheeks, perianal
and acral areas with growth retardation, alopecia and reddish tint of hair. He is likely to have a 77. An induration of > or = to 5 mm tuberculin skin test reaction is said to be positive if the following
deficiency of p 2248 condition/s is/are present in a child
A. Iodine p 962
B. Magnesium A. receiving corticosteroids of < or = to 10 mg per 24 hours for 2 weeks
C. zinc B. clinical or radiographic findings of TB disease
D. fluoride C. esposure to laryngotracheobronchitis
D. all of the above
64. A mother consulted the clinic to manage her 9 year old obese child. Her condition is most often
associated with the following features EXCEPT p 78. One of the following viral exanthems is difficult to recognize clinically as symptoms mimic other viral
176 infections as mild illness with lymphadenopathy, slight fever, concurrent with generalized
A. hyperpigmentation in the neck and skin creases erythematous maculopapular rash starting on the face
B. tall stature A. Roseola infantum
C. slightly advanced bone age B. Postnatal rubella
D. delayed onset of puberty C. Measles
D. Dengue fever
65. A child with non-edematous protein energy malnutrition is now on the last phase of therapy. Iron
therapy for this child is not given in the early phase of treatment because p 173 79.A 2 month old infant was brought to the clinic for immunization. Physical examination findings
A. free iron in the early phase may exacerbate oxidant damage were all within normal. Which of the following vaccines may be given?
B. iron does not promote growth in the early phase of treatment (0.25) A. DPT
B. Hepatitis A
C. Measles vaccine
D. Typhoid vaccine 93. Infections are a frequent and important cause of morbidity and mortality in the neonatal period
because:
80.A 3 year old girl was transfused with 3 units of fresh frozen plasma due to dengue A. Infectious agents can be transmitted from mother to the fetus or newborn infant by
hemorrhagic fever. When is the soonest time that MMR can be given after the transfusion? diverse modes.
A. > 3 months after B. Newborn infants are more capable of responding to infection because of less immunologic
B. > 6 months after deficiencies
C. > 9 months after C. Diagnosis and management of neonatal infections are not often complicated by coexisting
D. > 12 months after conditions
D. Maternal infections that is the source of transplacental fetal infection is often detected in
81.A 2 year old child was admitted under your service with a working diagnosis of measles to prevent early pregnancy.
further complications vitamin A must be given at a dose of p 1029
A. 50,000 IU 94. This is characteristic of a neonate with early onset of sepsis p 628
B. 100,000 IU A. Poor suck, hypothermia, hypoglycemia manifesting at 10 th day of life.
C. 150,000 IU B. Increased incidence of seizures and bulging fontanelles
D. 200,000 IU C. Multisystem involvement and higher incidence among preterm neonates 1 (1.0)

82. A 6 year old child and his family spent New Year in Baguio City. A high possibility of exposure to D. Uncommon maternal obstetric complications
meningococcemia prompted the mother to ask for possible chemoprophylaxis. The drug that can help
in such case is p 899 95. Vitamin K deficiency is characterized by
A. penicillin p 190
B. rifampicin A. low platelet count
C. erythromycin B. normal protime, abnormal PTT
D. chloroquine C. normal PTT and abnormal PT
D. abnormal PT and PTT
83. While doing a routine physical examination to a 4 months old infant you found out that his left testes is
undescended. As a preventive measure against the possibility of having cancer later in life, you advise 96. A prolonged PTT, normal PT and platelet count should make one suspicious of
the mother to have her child undergo orchiopexy if the testes will not descend before reaching the age of A. Acquired Prothrombin Complex Deficiency
B. Disseminated Intravascular Coagulopathy
C. Hemophilia
A. 5 -8 months old D. Idiopathic Thrombocytopenic Purpura
B. 9 15 months old
C. 2 4 years old 97. This is not usually administered in chronic Idiopathic thrombocytopenic purpura
D. 5 6 years old A. Vincristine sulfate
B. Danazol
84. If paralysis occurs in a 1 year old infant infected with poliovirus when is it expected to be observed? C. Steroids
D. Anthracycline
A. 14 days after onset of the illness
B. 1 day after onset of the illness 98. Treatment of choice for severe aplastic anemia is
C. 3 8 days after onset of the illness A. Antithymocyte globulin
D. 30 days after onset of the illness B. Steroids
C. Bone Marrow Transplant
85.Vaccine associated paralytic poliomyelitis D. Androgens
p 1039
A. occurs 7 14 days after giving inactivated polio virus vaccine 99. Severe joint pains, weight loss, organomegalies, anemia are features of
B. occurs 7 14 days after giving oral polio virus vaccine (0.5) A. Acute Lymphocytic Leukemia
C. occurs 15 30 days after OPV B. Aplastic anemia
D. occurs 15 30 days after IPV C. Fanconis anemia
D. von Willebrands disease
86.Ninety (90%) of viral meningitis cases in measles, mumps and german measles vaccinated children
has been shown to be caused by 100. Severe hemophilia will present as:
p 1045 A. Hemarthroses
A. Herpes simplex virus B. Petecchial rashes
B. Enteroviruses C. Epistaxis
C. Cytomegalovirus D. Ecchymoses
D. All of the above ---END---
87. A 1 year old male infant sought consultation for fever of 2 days with irritability/drooling of the saliva
and decrease formula/food intake. Oropharyngeal findings showed vesicles and ulcers on the
uvula, soft palate, anterior tonsillar pillars and posterior pharyngeal wall. The most likely diagnosis
would be
p 1045
A. Aphthous ulcers Subject: Pediatrics
B. Herpetic gingivostomatitis Instructions: Choose the BEST answer
C. Hand-foot and mouth disease
D. Herpangina 1. The parameters used to estimate the gestational age based on physical maturity are description
of the following EXCEPT:
88. Genital herpes virus infection occurs in A. lanugo
p 1053 B. plantar surface
A. any age group especially school age group C. genitalia
B. children via autoinoculation and sexual abuse (0.5) D. pupillary dilatation
C. primary and recurrent disease and can be differentiated clinically Ans: D
D. a neonate in 90% within the first week of life (0.5) 2. An infant weighing 1400 gm is born at 32 weeks gestation in a delivery room that has an ambient
temperature of 24 degrees centigrade. Within a few minutes of birth, this infant is likely to exhibit all
89. The management of an infant born to a pregnant woman with active genital HSV infection will consist the following EXCEPT:
of A. Pallor
A. Vaginal delivery for recurrent infection B. Shivering
B. Caesarean section within 4-hour of rupture of bag of water (0.33) C. a fall in body temperature
C. Infant should be treated immediately with acyclovir without need to perform cultures D. metabolic acidosis
D. If sign/s of HSV infection is/are present on the mother, a serologic Ans: B
determination of her immune status should be performed 3. The immediate postnatal changes in a term newborn includes the following EXCEPT
A. decrease in pulmonary vascular resistance
90.A 25 year old pregnant woman (+) for Hep Bs Ag delivered a term, AGA male newborn with an Apgar B. decrease in right to left shunting via ductus arteriosus
score of 9 (-1 for color) and 10 at 1 and 5 minutes. What measure will you undertake to prevent C. increase in venous return to the left atrium
perinatal infection? P 1329 D. increase right to left shunting via foramen ovale
A. Hepatitis B vaccination at 6 weeks, 10 weeks and 14 weeks of age Ans: D
B. Hepatitis B vaccination at 2 months, 4 months and 6 months of age 4. An infant has the following findings at 5 minutes of life; pulse 130 per minute, cyanotic hands and
C. Hepatitis B vaccination within 12 hours of life plus hyperimmune globulin against feet, good muscle tone, and a strong cry. This infants Apgar score is
Hepatitis B A. 7
D. Hepatitis B vaccination before 1 2 month of age B. 8
C. 9
91. Neonatal infants of diabetic mothers are at high risk for hypoglycemia because: D. 10
A. Hyperinsulinism exists Ans: C
5. A newborn infant was noted to have the following physical features at birth: Weight 3.0 kg, absent
B. There is impaired gluconeogenesis in response to hypoglycemia lanugo, white parchment-like desquamating skin and long nails. The infants gestational age is
C. Increased metabolic needs disproportionate to substrate stores and calories most likely :
supplied. A. >42 wks
D. All of the above B. 37 39 wks
C. <37 wks
D. 40-42 wks
92. Intractable neonatal hypoglycemia in infants with macroglossia, large size, visceromegaly, mild Ans: A
microcephaly, omphalocoele, facial nevus flammeus, a characteristic earlobe creases are seen in 6. Which of the following patterns noted on continuous monitoring of fetal heart rate is most indicative
infants with: of fetal distress?
A. Turner syndrome A. Baseline variability with periodic acceleration
B. Klinefelter syndrome B. Increasing baseline variability
C. Down syndrome C. Early deceleration pattern
D. Beckwith-Wiedemann Syndrome D. Late deceleration without baseline variability
Ans: D 22. A 6 month old child who weighed 3.0 kg at birth would have a weight of
7. Maternal condition that would post for high risk pregnancy A. 5 kg
A. Pregnancy interval of 2-3 years B. 7 kg
B. Maternal age 20-25 year old C. 9 kg
C. Primigravid status D. 11 kg
D. Inadequate pre-natal care
Ans: D Formula used:
8. Anticipated neonatal morbidities associated with maternal risk factors include
A. Neonatal macrosomia with Maternal Diabetes Age in months + 9
B. Neonatal euthyroid state with Maternal Graves disease ------------------------
C. Neonatal thrombocytosis with Maternal thrombocytopenia 2
D. Neonatal polycythemia with Maternal placenta previa
Ans: A 6+9 15
9. A mother delivers a neonate with meconium staining and low Apgar scores of 3 at 1 and 5 mins. -------- = ---------- = 7.5 kg
Of life. The goals of resuscitation are the following EXCEPT 2 2
A. Minimize heat loss
B. Establish normal respiration and lung expansion Ans: B
C. Support cardiac output 23. At age 15 months, which motor activity can be performed normally?
D. Anticipate neonatal demise A. Walks upstairs with alternating steps
Ans: D B. Climbs the stairs one step at a time
10. A newborn was noted to have recurrent episode of aspiration with excessive salivation, most likely C. Walks alone by herself
the mother had D. Runs and seldom falls
A. Polyhydramnios
B. Oligohydramnios Ans: C
C. Normohydramnios 24. At 12 months, a child can already
D. Anhydramnios A. Say bye-bye
Ans: A B. Say his full name
11. Which of the following blood factors are decreased in the newborn? C. Comprehend words as hello
A. VII, IX, and X D. Utter 2 words like the Dada and Mama
B. II, V, VII, and IX
C. V, VII, IX, and X Ans: D
D. VII, VII, IX, and XI 25. The first visible sign of puberty in boys.
Ans: B A. growth of pubic hair
12. Which situation is Jaundice most likely physiologic in a term infant? B. enlargement of the penis
A. Jaundice at 12 hours of age C. testicular enlargement
B. Serum bilirubin increasing by 5 mg/dl/24 hours or less in the first 2 to 4 days D. axillary perspiration
C. Direct (conjugated) serum bilirubin greater than 1 mg/dl
D. Jaundice at 12 days of age Ans: C
Ans: B 26. The sexual maturity of a girl whose breast buds appeared when she was 10 years old and had
13. The most appropriate treatment for hyperbilirubinemia (11.2 mg/dl) in a 3-week-old breast-fed menarche at 16 years old.
infant with normal growth and development? A. normal
A. Phototherapy B. precocious
B. Exchange transfusion C. delayed
C. Phenobarbital D. undetermined
D. None of the above
Ans: D Ans: C
14. A primiparous woman whose blood type is O-positive gives birth at term to an infant who has an A- If a girl has no breast buds by the age of 13 years, or if more than 5 years separate the
positive blood and a hematocrit of 55%. A serum bilirubin level obtained at 36 hours of age is 12 onset of pubertal change from menarche, her puberty is delayed.
mg/100 ml. Which of the following laboratory findings would be most characteristic of this infants
disease? 27. The most common stimulus in the gut provoking abdominal pain:
A. An elevated reticulocyte count A. edema
B. A weakly positive direct Coombs test B. ischemia
C. Nucleated red blood cells in the blood smear C. tension or stretching
D. Hematocrit less than 55% D. accumulation of tissue metabolites
Ans: D
15. A full term infant was born by normal spontaneous delivery. Nursery stay was unremarkable and Ans: C
baby was breastfed. On the 2nd week of life, baby was noted to be jaundiced. This baby is most 28. A 6 month-old male infant previously well, suddenly developed paroxysmal colicky abdominal pain
likely having: at frequent intervals accompanied by straining efforts is most likely having this condition.
A. Physiologic jaundice A. Acute Appendicitis
B. Pathologic jaundice B. Abdominal Epilepsy
C. Breast-milk jaundice C. Urinary Tract Infection
D. Kernicterus D. Intussusception
Ans: C
16. Typical abdominal x-ray in necrotizing enterocolitis: Ans: D
Paroxysmal colicky abdominal pain at frequent intervals in a previously well infant is
A. double-bubble sign
characteristic of Intussusception.
B. string sign
29. The most likely diagnosis of a 9 year old male with severe epigastric pain and muscle rigidity
C. Pneumatosis intestinalis
on the epigastrium, unrelieved by ordinary antispasmodic.
D. apple peel sign A. Urolithiasis
Ans: C
B. Abdominal Epilepsy
17. A fecaloid vomitus indicates
C. Acute Pancreatitis
A. Obstruction to the stomach
D. Acute Hepatitis
B. Obstruction in the duodenum proximal to the ampulla of Vater
Ans: C
C. Narrowing or closure of the intestinal lumen distal to the ampulla of Vater
The pain in Urolithiasis may also be severe but is usually located at the lumbar and lower
D. Obstruction low in the intestinal tract back areas. The pain in abdominal epilepsy is usually in the periumbilical area. Abdominal
Ans: D pain in Acute Hepatitis is usually not severe. Acute Pancreatitis is characterized by
18. TRUE about necrotizing enterocolitis: severe epigastric pain and muscle rigidity.
A. It is primarily a disease of infants 6-12 months old.
B. Breast milk feedings are not protective. 30. The recommended daily allowance of Protein during the 1st 6 months of life is about:
C. Bloody stools are seen in most patients. A. 1.0 g/kg/24 hr
D. It usually presents with abdominal distention with gastric retention. B. 2.0 g/kg/24 hr
Ans: D C. 3.0 g/kg/24 hr
19. A two-week-old premature infant is found to have several milliliters of formula still present is the D. 4.0 g/kg/24 hr
stomach two hours after being fed. Also noted are gastric distention and the passage of blood-
streaked stools. Which historical factor would best support a tentative diagnosis of necrotizing Ans: B
enterocolitis? The current RDA for protein during the first 6 months of life in a healthy term infant is 2.0-2.2
A. Passage of a thick tenacious meconium plug at 24 hours of age g/kg/24 hour.
B. Severe hyaline membrane disease with anoxic episodes in the first week of life 31. Breastmilk from mothers whose diet is sufficient and balanced supply all the necessary nutrients
C. A maternal history of severe ulcerative colitis needed by the infant. One of the following is found to be insufficient at birth:
D. A history of milk-protein allergy in family members A. Vitamin A
Ans: B B. Vitamin D
20. A woman was noted to have a large volume of amniotic fluid at the time of her delivery of her child. C. Fluoride
At 6 hours of age, her baby begins regurgitating small amounts of mucus and bile-stained fluid. D. Iron
P.E. is normal. Abdominal x-ray obtained showed a double-bubble sign. The most likely
diagnosis of this infants disorder: Ans: C
A. Esophageal atresia Amount of iron and Vitamin D are sufficient during the 1st 4-6 months of life. Supplement is needed
B. Pyloric stenosis beyond this period. If the water supply is not adequately fluoridated (< 0.3ppm), the breastfed infant
C. Midgut volvulus should receive at least 10 ug of fluoride daily for the 1st 6 months of life.
D. Duodenal atresia
Ans: D 32. Most normal newborns are thought to have sufficient stores of iron. However, iron deficiency is still
21. A 1 month old infant forces at near object attains its visual activity of 20/20 at age a common problem during infancy. One of the following statement is true regarding iron deficiency
A. 2 in infants:
B. 3 A. Human milk contains less iron than most formulas, thus breastfed infants are prone
C. 4 to develop anemia than formula fed infants.
D. 5 B. Amount of iron stores at birth and its absorption are variable, thus onset of iron
Ans: C deficiency may also vary.
C. Inadequate dietary intake of iron is the most common cause of this problem. C. Phenylketonuria
D. Rapid destruction of RBC during infancy D. Oast house urine disease
Ans: C
Ans: B
Although human milk contains less iron than most formulas iron is absorbed 2-3x more 45. A 2 week old baby boy, delivered NSD, TERM, AGA, was admitted due to poor suck and vomiting,
efficiently from human milk than from cows milk. Iron deficiency secondary to inadequate upon admission patient was stuporous, dehydrated with sweet smelling urine. The most likely
dietary intake is not common before 6 months of life. diagnosis:
33. An otherwise healthy 6 month old infant was brought to your clinic because of restlessness, crying A. Penylketenuria
and failure to gain weight. What is the possible cause of the childs problem: B. Oast house urine disease
A. This infant is suffering from septicemia C. Maple syrup urine disease
B. Late introduction of complementary feeding D. Xanthunuria
C. Possibility of an abnormal mother-infant bonding
D. All of the above Ans: C
46. Lipid Storage disease associated with the deficiency of the lysosomal enzyme B- hexosaminidase
Ans: D A is:
Underfeeding is suggested by restlessness, crying, slow weight gain or actual weight l A. Gaucher Disease
oss. All of the above can cause infants failure to take sufficient quantity of food. B. Lesch-Nyhan Disease
C. Niemann-Pick Disease
34. The most common cause of death from physical abuse is: D. Tay-Sachs Disease
A. Intra-abdominal injuries
B. Burns Ans: D
C. Intentional Head Trauma (IHT) 47. Jesus, a 7 year old 2nd grader was brought by her mother to their physician because he was
D. Poisoning always in trouble at school and impossible at home. He cannot sit still at the dinner table or
anywhere else, except perhaps in front of the television. Physical examination revealed nothing
Ans: C more than a slight difficulty in performing skillful motor acts and some clumsiness. The most likely
More than 95% of serious intracranial injuries during the 1st year of life are the result of diagnosis is:
IHT. Intra-abdominal injuries from impacts are the second most common cause of death A. Temporal lobe epilepsy
in battered children. B. Conduct disorder
35. Bruises are the most common manifestations of child abuse. Appropriate age of bruises allows C. Attention deficit hyperactivity disease
physicians to correlate history with actual age of injury. A bruise that is characterized as purple or D. Attention deficit hyperactivity disorder
bluish is approximately:
A. 0-48 hours Ans: D
B. 48-72 hours 48. An 18 year old farm worker was brought to the ER with symptoms of abdominal cramps, excessive
C. 4-7 days salivation, vomiting diarrhea and muscle fasciculation. History revealed that he has been applying
D. > 7 days pesticides on a large area of the banana plantation where he worked. The most common cause of
his problem:
Ans: B A. Dioxin poisoning
Bruise that is fresh (0-48 hours) is red; purple-blue is 48-72 hours; yellow-green 4-7 days; brown > B. Organophosphate poisoning
7 days. C. Hydrocarbon poisoning
36. A 2 year old male child was seen at the ER due to 2nd degree burns of both hands. What would you D. Heavy metal poisoning
do?
A. Admit the patient since you cannot clearly tell if the burns are intentional or not Ans: B
B. Treat the burns and send home the patient 49. A 20 year old G1P0 woman gave birth to a baby boy with Down Syndrome. Her first pregnancy
C. Refer to the surgeon on duty for management of the burns resulted in abortion. Such chromosomal abnormality is usually due to:
D. Ask for the immunization status of the patient A. Translocation
B. Nondisjunction
Ans: A C. Mosaicism
In cases where the diagnosis is unclear, always admit the patient. The parents should be D. Point mutation
told why an inflicted injury is suspected.
37. The use of helmets, seat belts, knee and elbow pads are interventions that: Ans: A
A. prevent the occurrence of the injury-producing agent 50. A 12 year old female was brought in for consultation because of poor performance in school
B. will attempt to reduce the likelihood of injury by modifying the transfer of energy to the especially in mathematics. On physical examination she was found to have short stature, low
victim posterior hairline, webbed neck, widely spaced nipples and sexual infantilism. The most likely
C. will limit the impact of injuries on the victims diagnosis:
D. all of the above A. Klinefelter Syndrome
Ans: B B. Turner Syndrome
The use of seat belts and other safety devices will modify the rate of release of the hazard C. Edward Syndrome
from its source. D. Adrenogenital Syndrome
38. A reliable index of long-term glycemic control is provided by measurement of :
A. FBS Ans: B
B. Urine sugar 51. Which of the following organisms is the major cause of severe systemic and focal infections in
C. Glycosylated hemoglobin newborns?
D. Oral glucose tolerance test A. Staphylococcus aureus
Ans: C B. Streptococcus viridans
39. The first and rate limiting step in steroid synthesis is the conversion of cholesterol to: C. Group B streptococcus
Aldosterone D. Pseudomonas species
A. Cortisol
B. Pregnenolone Ans: C
C. sex steroids 52. A four year old boy was brought to your clinic because he was exposed to his grandmother who
Ans: C was coughing out blood for two months already. He has good weight gain and appetite, and has
40. At what age is onset of puberty considered precocious in girls? no chronic cough nor fever. You administer a Mantoux tuberculin test and the reading after 72
A. < 7 years of age hours is 15mm. What category does this child belong to?
B. < 8 years of age A. TB Exposure
C. < 9 years of age B. TB Infection
D. < 10 years of age C. TB Disease
Ans: B D. TB Inactive

41. A newborn with congenital Hypothyroidism is diagnosed and treated adequately by two weeks of Ans: B
age. It can be anticipated that with continued treatment he will have: 53. A 5 year old girl developed fever, coryza and conjunctivitis. After 5 days, still with fever, she was
A. Short stature but normal intelligence noted to have maculo-papular rashes and cervical lymphadenopathy. What is your diagnosis?
B. Short stature and slight mental retardation A. Rubella
C. Normal growth and development B. Roseola
D. Normal stature but a mild degree of mental retardation C. Fifths Disease
D. Rubeola
Ans: C
Ans: D
42. A 1 year and 6 months old child has congenital adrenal hyperplasia. He carries a boys name, and 54. A mother with a known herpes simplex type 2 infection gave birth vaginally to a full term healthy
had been raised as a boy. P.E shows a moderately enlarged phallus but empty scrotal sacs. looking baby boy with the assistance of a hilot. What is the next immediate step to do after
Chromosome analysis shows an XX pattern. If you were the doctor, you would: resuscitation?
A. continue to raise him as a boy A. Give herpes simplex vaccine intramuscularly within first 12 hours of life
B. ask parents how they feel and follow their wishes B. Send baby to the hospital for admission and intravenous acyclovir treatment
C. advise the parents that it would be best to raise the child as a girl after surgical C. Advise mother to closely observe baby for appearance of oral lesions
correction of the external genitalia D. Start first dose of oral acyclovir and refer to a pediatrician for further management
D. wait until the child is old enough to decide which sex to assume
Ans: B
Ans: C 55. People of all ages can develop acute rheumatic fever, but it usually occurs in children
A. 0 1 year old
43. The most severe form of mucopolysaccharidoses due to the deficiency of alpha-L- iduronidase is: B. 2 4 years old
A. Hurlers syndrome C. 5 15 years old
B. Hunters syndrome D. 16 19 years old
C. Morquios syndrome
D. Sanfillippo syndrome Ans: C
Ans: A 56. The diagnosis of acute rheumatic fever is mainly determined by
44. Inborn error of amino acid metabolism associated with mousy or musty odor of the urine is: A. blood culture
A. Tyrosinemia B. clinical signs
B. Hawkinsinuria C. electrocardiogram
D. acute phase reactants B. Infection
C. Hemorrhage
Ans: B D. Immunodeficiency
57. NOT TRUE regarding mitral stenosis: Ans: A
A. It is usually rheumatic in origin seen in older children and adolescents 70. A 4 year old child with brain tumor underwent radiotherapy and chemotherapy. After 2 years, the
B. It may cause concentric hypertrophy of the LV child was noted to be stunted (height at p35). What is the likely reason for such a finding?
C. The heart murmur is diastolic in timing A. Decrease in brain size due to radiotherapy lessened capacity of growth
D. It can be managed with balloon valvuloplasty hormones to induce growth
B. Damage to hypothalamic axis has decreased amount of growth hormones
Ans: B causing stunting
C. Chemotherapy caused hypoperfusion and ischemia of the brain
58. A 7 year old girl presents with a tender and swollen right knee as well as a more recently appearing D. Radiation induced hypoplasia of the skull which limited brain development
swollen left ankle. She also has fever. Which of the following modified Jones criteria does the Ans: B
patient fulfill? 71. Laboratory test to confirm the diagnosis of neuroblastoma:
A. 1 Major 1 minor A. Urine vanillylmandelic acid (VMA)
B. B.1 Major 2 minors B. Urine catecholamins
C. C.2 Majors C. Serum alpha-fetoproteins (AFP)
D. D. 2 Minors D. Serum beta- human chorionic gonadotrophin (B-HCG)

Ans: A Ans: A
59. The patient is 10 year old who had frequent tonsillitis and he had complained of migratory joint
swelling, intermittent high grade fever, palpitations and easy fatigue. PE includes BP 180/20 CR 72. A 2 year old child was brought to the emergency room for on and off fever and easy bruisability for
110/min. Heart dynamic precordium, AB at 6th LICS, AAL, S1 normal, S2 split P2 accentuated, 4 weeks already. On PE, you noted palmar pallor, generalized lympadenopathy and
Grade 3/6 diastolic blowing murmur LUSB radiating to apex, Peripheral pulses bounding. The hepatomegaly. What is the most likely cause for his signs & symptoms?
diagnosis is that he has RHD. What is the most likely cardiac pathology? A. Connective tissue disease
A. mitral insufficiency B. Chronic infection
B. mitral stenosis C. Blood malignancy
C. aortic insufficiency D. Nutritional anemia
D. aortic stenosis
Ans: C
Ans: C
60. What is the most common cause of asthma attacks in the infancy period? 73. A 5-year-old boy was brought to the OPD for frequent vomiting and abdominal distention. He has
A. Viral infection had poor appetite for almost one month and would complain of constipation and reddish urine. On
B. Stressful activity PE, BP= 130/90, T= 38.2C. You palpated a mass on the right side of the abdomen which was firm,
C. Exposure to allergen not movable and non-tender. What is your diagnosis?
D. Food hypersensitivity A. Rhabdomyosarcoma
B. Hirschsprungs disease
Ans: A C. Impacted feces
61. During the first year of life, the most common scenario in a child with adverse reaction to foods is: D. Wilms tumor
A. Rashes after intake of soy-based foods
B. Drowsiness after intake of cured meat Ans: D
C. Pruritic erythematous patches after intake of cheese
D. Watery, blood-streaked stools after cows milk intake 74. The most common cause of bacterial meningitis at 2 months of age is:
A. Group B Streptococcus
Ans: D B. Mycoplasma pneumoniae
62. A preschooler was brought to the ER for difficulty of breathing. She was noted to be breathless C. Psuedomonas aeruginosa
while talking in phrases and prefers to sit when examined. On PE, her RR is >40/minute, D. Listeria monocytogenes
wheezing was heard throughout expiration, was tachycardic and PEFR was 55%. What is the
classification of severity of this childs acute asthma attack? Ans: A
A. Mild intermittent 75. Lumbar puncture is indicated in the following conditions:
B. Mild persistent A. Child with encephalitis with signs of increased intracranial pressure
C. Moderate persistent B. Child who present with headache, associated with left-sided hemiparesis, facial
D. Severe persistent asymmetry and ptosis
C. Child who presents with fever, generalized seizures and nuchal rigidity
Ans: C D. Child with fever, seizures, purpuric rash and hematoma at all puncture sites

63. A 7-year-old boy was brought to the ER due to tightness in the chest. Earlier, he mentioned that he Ans: C
was stung by a bee. Around 30 minutes after the sting, he felt warm and had difficulty swallowing 76. A 10-year-old boy developed severe headache, photophobia and vomiting. He is febrile.
his saliva. He started feel dizzy and was noted to have difficulty of breathing. On PE he was noted P=180/100 mm Hg. Results of cranial nerve and motor examinations are normal. He has nuchal
to have a BP of 50 palpatory, HR of 50/min regular in rhythm, RR of 50/min, with wheezing all over rigidity and extensor plantar response, (+) Babinski. The most appropriate to obtain this time is:
his lung fields. What is the immediate treatment of choice for this patient? A. CSF analysis
A. Oral corticosteroid B. Electroencephalogram
B. Inhaled beta-2 agonist C. CT scan
C. Intramuscular epinephrine D. Skull x-ray
D. Oral antihistamine
Ans: C
Ans: C 77. You are evaluating a 5-year-old girl who has fever, vomiting, and nuchal rigidity. CSF examination
64. Based on the National Prevalence Survey, how many mm induration response to PPD 5 TU test, reveals WBC=1650, 85% segmenters, 15% lymphocytes, glucose=20 mg/dl and protein=250
differentiate the TB infected from the non infected Filipino? mg/dl. Gram stain showed Neisseria meningitides. The best choice of parenteral antibiotic for this
A. 5 patient is:
B. 8 A. Penicillin
C. 10 B. Ceftriaxone
D. 15 C. Nafcillin
Ans: C D. Vancomycin
65. A 2 year old boy has recurrent breathing pauses and snoring during sleep, consulted at the ER due
to difficulty of breathing. On PE his tonsils are swollen and enlarged. Your most commonly Ans: B
diagnosis is? 78. A 7 year old child was admitted for Acute Post Strep Glomerulonephritis. What is the usual
A. Acute epiglottitis presenting symptom of the patient
B. Acute tracheitis A. Headache
C. Obstructive sleep apnea B. Dysuria
D. Diptheria C. Periorbital edema
Ans: C D. Abdominal Pain
66. A 6 month old male infant has mild upper respiratory tract infection and low grade fever gradually
develops into respiratory distress characterized by wheezing and dyspnea. The most likely Ans: C
diagnosis is? 79. Anemia in Post Strep Glomerulonephritis is due to:
A. Bronchial asthma A. High grade hemoglobin
B. Acute bronchiolitis B. Hematuria
C. Acute bronchitis C. Hemodilution
D. Laryngotracheobronchitis (LTB) D. decrease production of RBC
Ans: B
67. NOT included in the management of Acute Laryngotracheobronchitis: Ans: C
A. Give O2 and supportive care 80. Which of the following indicate that the patient is experiencing a severe complication of acute
B. Start racemic epinephrine by aerosol glomerulonephritis?
C. Add inhaled steroids A. Temperature of 38.8 C
D. Admit and start cephalosporins B. Blood Pressure of 140/92 /mm Hg
Ans: D C. Severe Sodium of 140 mg/ L
D. Weight loss of 2 lbs
68. A 2 year old with class III childhood TB would have the following characteristics EXCEPT:
A. (+) history of exposure to an adult with active TB Ans: B
B. (+) mantoux tuberculin test 81. A 3 year old boy come in the ER with abdominal pain generalized edema, BP 90/60 mmHg T36.8C
C. abnormal chest radiographs suggestive of PTB HR85/min BUN 217mg/dl Cr).5 mg/dl, Urine Protein is1800mg/24h. Impression is:
D. Started on INH, Rifampicin, PZA and Ethembutol A. Congestive Heart Failure
Ans: D B. Idiopathic Nephrotic Syndrome
69. What is the main stimulus for red blood cell production? C. Systemic Lupus Erythematosus
A. Anoxia D. Post-streptococcal Glomerulonephritis
B. muscle cramps
Ans: B C. somnolence
82. The most common etiologic agent involved in acute hematogenous osteomyelitits in children is D. tachycardia
A. Group B streptococcus
B. Group A streptococcus Ans: D
C. Staphylococcus aureus 96. Normal plasma osmolality is approximately
D. Hemophilus influenza type B A. 265-275 mOsm/kg H20
B. 285-295 mOsm/kg H20
Ans: C C. 305-315 mOsm/kg H20
83. Which of the following statements is TRUE? D. 325-335 mOsm/kg H20
A. Septic arthritis is a disease most commonly found in adolescent males
B. In septic arthritis, the hips and knees are the most commonly affected joints Ans: B
C. In a child with septic arthritis of the hip, redness swelling and warmth are often 97. Hyponatremia defined as <130 mEq/L of sodium may be caused by the following EXCEPT:
detectable on PE A. Mineralocorticoid deficiency
D. Children with transient synovitis never present with fever B. Osmotic diuresis
Ans: B C. Symptom of inappropriate ADH secretion (SIADH)
84. Cardinal features of Ehlers-Danlos syndrome include all of the following EXCEPT: D. Diabetes insipidus
A. hyperextensible doughy skin
B. palpable purpura Ans: D
C. joint hypermobility
D. vascular fragility and brusing 98. The following are consequences of hypokalemia EXCEPT:
Ans: B A. Paralytic ileus
85. These are group of bone diseases in which the ossification centers undergo avascular necrosis B. Prolonged QT interval
followed by resorption, fragmentation of dead bones and finally regeneration and replacement of C. Increased neuromuscular excitability
reparative bone tissues. D. Weakness
A. Osteochondroma
B. Osteochondroses Ans: C
C. Osteopetrosis 99. A 3 year old child with diarrhea who presented with lethargy, rapid feeble pulses and very sunken
D. Osteochondritis eyeballs has an estimated fluid deficit of:
Ans: B A. 30-50 ml/kg
86. The most feared complication of Kawasaki Disease: B. 60-90 m/kg
A. Pericarditis C. 100 ml/kg
B. Coronary Aneurysm D. 150 ml/kg
C. Aseptic Meningitis
D. Myocarditis Ans: C
Ans: B 100. An arterial blood gas determination taken on room air showing the following values pH= 7.3,
87. A child who has low grade fever, abdominal pain, arthritis, microscopic hematuria and purpuric rash pCO2= 35 mmHg, HCO3= 16 mmol/L, PO2= 60 mmHg is indicative of:
only in the lower extremity. He most likely has A. Metabolic acidosis and normal oxygenation
A. Meningococcemia B. Metabolic acidosis and hypoxemia
B. Poststreptococcal Glomerulonephritis C. Respiratory acidosis and hypoxemia
C. Henoch-Schoenlein purpura D. Normal acid-base balance with hypoxemia
D. Wegeners granulomatosis
Ans: C Ans: B
88. In the diagnosis of Marfan syndrome, the most useful information is
A. History of consanguinity
B. Chromosomal mapping to detect deletions of Chromosome 15
C. Width of aortic root exceeding more than 95th percentile CEBU INSTITUTE OF MEDICINE
D. Measurement of fibrillin precursors in peripheral leucocytes
Ans: C PEDIATRICS
89. A 13 year old girl is seen because of 4 weeks history of body malaise, anorexia, intermittent fever
and arthritis of ankles, wrist and knees. She developed shortness of breath and dyspnea on 1. Which of the following findings is NOT consistent with physical abuse?
exertion. PE revealed BP 92/58, RR of 24, Pulse 125, crackleson both lung bases, muffled heart A. Implement marks on the torso
sounds accompanied by friction rub. 2 D Echocardiography documents pericardial effusion. The B. Bruises of varying ages
most useful screening laboratory test for this patient is C. Symmetric bruises
A. Serum Ig levels D. Bruises on the shins, knees and elbows
B. Antinuclear antibody (Answer: D / Reference: A p 1125 / MPL: 0.6)
C. Serum complement levels
D. Human lymphocyte antigen (HLA) 2. Which of the following is the best way to manage dermal exposure to a potentially toxic substance?
A. Wash the exposed area with copious amounts of neutralizing solution
Ans: C B. Wash the exposed are with lots of soap and water
90. A 14 year old male is referred for evaluation of heart murmur noted on sports pre-participation C. Wash the exposed area with copious amounts of lukewarm water
physicals. PE reveals a very tall thin boy who has pectus deformity of the chest, hyperextensible D. Give oral antidote
joints, and apical holosystolic murmur with mid diastolic rumble. The most helpful diagnostic (Answer: C / Reference: A p 2364 / MPL: 0.6)
evaluation to determine the cause of these findings is
A. Skeletal survey 3. A 3 year old female was brought to your house because of vomiting, abdominal pain, and nausea.
B. Slit lamp eye examination While examining her, you noted that she had a garlic smell emanating from her mouth. What will you do?
C. Measurement of plasma amino acids A. Induce emesis
D. MRI of the spine and sternum B. Give 6-8 egg whites
Ans: B C. Call an ambulance and bring her to the hospital immediately
91. Which of the following drugs is contraindicated for use by the breastfeeding mother? D. Give activated charcoal
(Answer: C / Reference: A p 1426 / MPL: 0.33)
A. Ergotamine
B. Carbamazepine
4. Which of the following statements is NOT TRUE regarding nephrotic Syndrome?
C. Phenytoin
A. elevated serum triglycerides
D. Valproic acid B. may cause iron deficiency anemia
Ans: A C. normal ionized calcium
92. A 2500 gram infant who is born at 36 weeks AOG has a head circumference of 27 cm and crown- D. may have decreased intravascular volume
heel length of 40 cm. Other findings include upturned nose, hypotonia, hypoplastic philtrum. The (Answer: C / Reference: A p 1755 / MPL: 0.25)
most likely prenatal agent that would explain these findings is
A. Alcohol 5. Which of the following is NOT a typical course for acute post-streptococcal glomerulonephritis?
B. Cocaine A. Improvement of the nephritic manifestations can be observed after 1 week of onset
C. Marijuana B. Hypocomplementenemia resolves after 6 months
D. Opiates C. Gross hematuria resolves after 2 months
Ans: A D. Microscopic hematuria may persist for a year
93. Physical examination of a newborn female infant reveals meningomyelocoele, cleft lip and craniofacial(Answer: B / Reference: A p 1740 / MPL: 0.25)
anomalies. The most likely prenatal experience to explain these findings is
A. alcohol 6. Which of the following is NOT TRUE regarding the Hemolytic Uremic Syndrome?
A. Associated with thrombocytopenia
B. Lithium B. Associated with hematuria
C. Responds to steroid therapy
C. Thiazides D. Can be recurrent
(Answer: C / Reference: B p 1746 - 1747 / MPL: 0.33)
D. Valproic acid
Ans: B 7. Which substance is being tested in the Philippine Newborn Screening for Congenital Adrenal
94. A 5 year old boy who is taking Carbamazepine for control of generalized seizure presents with Hyperplasia?
signs and symptoms of pneumonia. Which of the following antibiotics most likely cause toxicity if A. Progesterone
prescribed concomitantly with Carbamazepine? B. 17 OH pregnenolone
A. Ampicillin C. !7 OH progesterone
B. Erythromycin D. 11 B hydroxylase
C. Vancomycin (Answer: C / Reference: B 1912 / MPL: 0.6)
D. Trimethoprim-Sulfamethoxazole
Ans: B 8. Which of the following syndromes is associated with elevated growth hormone levels?
95. A 15 year old patient has asthma and is taking salbutamol or albuterol via metered dose inhaler. A. Sotos syndrome
You are asked the known side effects of this drug by this patient. The most common side effect to B. Laron syndrome
tell is C. Klinefelters syndrome
A. Seizure D. Marfans syndrome
(Answer: B / Reference: B p. 1848 / MPL: 0.25)
9. Which of the following is the drug of choice for complex partial seizures? 21. Which of the following test would you order next?
A. Carbamazepine A. Glucose 6 phosphate dehydrogenase assay
B. Phenytoin B. Hemoglobin electrophoresis
C. Phenobarbital C. Hold breastfeeding and give milk formula
D. Diazepam D. Observation and reassure the mother
(Answer: A / Reference: B p. 2003 / MPL: 0.9) (Answer: A / Reference: B pp 1636 - 1638 / MPL: 0.33)

10. A 700 gm infant presents with abdominal distention, excessive gastric residuals and bloody stools. 22. A 6 hour old term infant delivered spontaneous cephalic was noted to have a swelling over the parietal
Physical examination reveals lethargy, poor perfusion and recurrent apnea. Abdominal examination area which does not cross the suture lines. The swelling pulsates and becomes tense on crying. Which of
reveals tenderness, guarding and erythema. You suspect necrotizing enterocolitis (NEC). Of the following, the following is the most likely diagnosis?
the MOST accurate statement about NEC is that: A. Caput succedaneum
A. age at onset is inversely related to gestational age at birth B. Cephalhematoma
B. it is more common among critically ill than convalescing neonates C. Cranial meningocele
C. pneumatosis intestinalis is the earliest radiographic sign D. Molding
D. strictures that occur as a late complication appear most frequently in the ileum (Answer: C / Reference: B 1985 - 1986 / MPL: 0.9)
(Answer: A / Reference: B pp. 590 - 591 / MPL: 0.6)
B23. Which of the following is the recommended vaccination schedule for neonates born to HBsAg
11. A one month old infant was noted to have a small fontanel. Presence of persistently small fontanel is positive mothers?
seen in all of the following conditions, EXCEPT: A. Hepatitis B immunoglobulin vaccine within the 12 hours after birth, Hepatitis B
A. cranio synostosis vaccine at 1 month, 3 months and 6 months old
B. congenital hyperthyroidism B. Hepatitis B Immunoglobulin vaccine at birth and Hepatitis B vaccine at birth, 1 month
C. congenital rubella syndrome and 6 months old
D. wormian bones C. Hepatitis B vaccine at birth, one and six months old
(Answer: C / Reference: B p. 525 / MPL: 0.33) D. None since mother is not infectious
(Answer: B / Reference: B pp 1328 - 1929 / MPL: 0.6)
12. An infant is born full term to a woman who had an uncomplicated pregnancy. Immediately after
delivery, the infant has severe respiratory failure. Breath sounds are diminished bilaterally. The abdomen For questions No 24-27. A 2-year-old male infant was noted to have intermittent episodes of cyanosis. He
is flat. The CXR shows a multicystic mass in the left chest with a shift of the mediastinum to the right. was noted to assume a squatting position while playing. The mother claimed he was not a blue baby at
What is the most likely diagnosis? birth and he has no recurrent episodes of cough.
A. Congenital lobar emphysema
B. Diaphragmatic hernia 24. A chest x-ray of the patient was done. Which of the following finding would be seen in this patient?
C. Respiratory distress syndrome A. Boot shaped heart
D. Persistence of fetal circulation B. Normal heart but with increased pulmonary vascularity
(Answer: B / Reference: B pp. 1353 - 1355 / MPL: 0.6) C. Heart described as egg on the side
D. Left ventricular hypertrophy
13. A mother is first seen at 28 weeks gestation with severe preeclampsia. A decision is made to deliver (Answer: A / Reference: B pp 1524 - 1528 / MPL: 0.9)
the infant. At birth the infant is placed in 100% oxygen. The first arterial blood gas reveals that the infant is
hypoxic and has elevated PCO2. Which of the following immediate diagnostic and therapeutic plans is 25. Which of the following is true regarding the above patient?
considered the least priority? A. Patient is prone to cerebrovascular event secondary to thrombosis
A. chest radiograph B. Congestive heart failure is a frequent complication
B. exogenous surfactant C. The problem will resolve spontaneously at 5 years of age
C. provide mechanical ventilation D. Bounding pulses is a characteristic feature
D. provide volume expansion (Answer: A / Reference: B pp 1524 - 1528 / MPL: 0.6)
(Answer: D / Reference: B pp. 570 - 572 / MPL: 0.6)
26. An electrocardiogram was done on the patient. Which of the following findings would be true in this
14. An infant born at 30 weeks gestation begins to experience apnea on the second day of life. Which of patient?
the following is the least appropriate initial management option for this infant? A. Dominant R waves in the right precordial chest leads
A. therapy with theophylline B. Left axis deviation
B. evaluation of evidence of hypoxia, infection or intracranial hemorrhage C. Negative T waves in V3R and V1 leads
C. CBC, arterial blood gas studies, plasma glucose and electrolyte measurement D. Peaked R waves in lead II
D. Check that the environmental temperature is in the neutral thermal zone (Answer: A / Reference: B pp 1524 - 1528 / MPL: 0.33)
(Answer: A / Reference: B pp. 573 - 574 / MPL: 0.33)
27. Which of the following drugs is useful in preventing hypoxic spells in the above patient?
15. An infant is born 28 weeks and weighs 1028 grams. The infant is started on enteric feedings at 3 days A. Aspirin C. Captopril
of age. Three days later, he is not tolerating his feedings. Which of the following signs and symptoms B. Dopamine D. Propanolol
exhibited by the infant is the least specific for neonatal NEC? (Answer: D / Reference: B pp 1524 - 1528 / MPL: 0.6)
A. bile-stained gastric fluid
B. pneumatosis intestinalis Questions 28-29. While at school a 6 year old boy is noted by his teacher to experience 10-20 second
C. abdominal distention lapses in consciousness, sometimes with clonic movements of the face. His parents have not noticed this
D. jaundice behavior at home. EEG shows 3 per second generalized spike and wave discharges.
(Answer: D / Reference: B pp. 590 - 591 / MPL: 0.6)
28. What is the MOST likely diagnosis of this boy?
16. An infant was born at 26 weeks gestation. The infant is now 8 weeks old and has broncho-pulmonary A. absence seizures C. attention deficit
dysplasia and retinopathy of prematurity (ROP). Which of the following statements is NOT TRUE of the disorder
pathogenesis and treatment of ROP? B. complex partial seizures D. generalized tonic clonic seizure
A. its development is related to retinal vessel immaturity and hyperoxia. (Answer: A / Reference: B pp. 1997 / MPL: 0.6)
B. Cryotherapy is the treatment of choice for all stages of disease.
C. The retinopathy resolves spontaneously in most infants. 29. Which of the following drugs is the MOST appropriate therapy for this boy?
D. Myopia is a common sequelae. A. carbamazepine C. gabapentin
(Answer: B / Reference: B pp. 2113 - 2114 / MPL: 0.33) B. ketogenic diet D. valproic
acid
17. An infant in the delivery room is noted to have respiratory movements, but no air is entering the lungs (Answer: D / Reference: B pp 2003 / MPL: 0.6)
with the mouth closed. Which of the following is the MOST likely diagnosis?
A. narcosis Question Nos. 30-32. A 1-month-old infant was admitted because of cough and respiratory distress.
B. choanal atresia Pertinent PE findings include rales on both lung fields and a loud continuous murmur. The apical impulse
C. pulmonary hypoplasia was also noted to be prominent. Pulses were strong. Mother claimed there was no history of cyanosis but
D. congenital heart disease she noted that the baby tires easily during feeding.
(Answer: B / Reference: B 1386 - 1387 / MPL: 0.9)
30. The chest x-ray of the above patient will most likely show which finding?
18. Jaundice is most likely physiologic in a term infant in which one of the following situations? A. left to right shunting with increased pulmonary vascularity
A. jaundice at 12 hrs of age B. prominent pulmonary artery with increased pulmonary vascularity
B. serum bilirubin level increasing less than 5mg/dL/day in the first 2 to 4 days C. normal heart with hyperaeration of both lung fields
C. jaundice at 12 days of age D. right ventricular hypertrophy
D. direct serum bilirubin greater than 1mg/dL (Answer: A / Reference: B pp. 1520 - 1512 / MPL: 0.6)
(Answer: B / Reference: B 594 - 595 / MPL: 0.6)
B31. Which of the following statements is true in this patient?
C19. A 900 gm infant of 27 weeks gestation developed respiratory distress syndrome and required A. Right atrial hypertrophy is a most likely consequence
endotracheal intubation on the 1st day of life. At 36hours of age, the infant developed hypotension, B. Blood from the aorta is shunted to the pulmonary artery
bradycardia, cyanosis and tense anterior fontanel. The most appropriate diagnostic test is: C. Brain abscess is a frequent complication
A. EEG D. Pressure in the right ventricle is usually decreased
B. Echocardiography (Answer: B / Reference: B pp 1510 - 1512 / MPL: 0.6)
C. Ultrasound of the head
D. Serum coagulation profile 32. Which of the following management is the most applicable in this patient?
(Answer: C / Reference: B pp. 562 - 564 / MPL: 0.6) A. Surgical closure before 1 year of age
B. Pharmacologic therapy with indomethacin
For Questions 20 to 21: A 3-day-old male neonate was noted to have pallor and jaundice. The baby was C. Control the heart failure and do surgical closure at 7 years old
otherwise active, with good suck, good urine output. Stools were greenish black in color. There was no D. Surgical closure at 2 years old if theres no spontaneous closure
hepatosplenomegaly. The mothers blood type is O Rh+ and the babys blood type is also O Rh +. (Answer: A / Reference: B pp 1510 - 1512 / MPL: 0.33)
Coombs test was negative. Hemoglobin was 11 g/dL and reticulocyte count was 7%. The smear showed
normocytic normochromic red cells with macrocytes noted. 33. A previously healthy and developmentally normal 18 month old boy presents with status epilepticus.
There is no past or family history of seizures. The child is stabilized and the seizure is halted. Subsequent
20. What is the most likely diagnosis? PE reveals a playful child who has a temperature of 39.2C and nasal discharge. Findings on the
A. Alpha Thalassemia major remainder of the examination are normal. Which of the following is the most likely cause of the childs
B. Breastfeeding jaundice status epilepticus?
C. Glucose 6 phosphate dehydrogenase deficiency A. bacterial meningitis C. fever
D. Physiologic jaundice B. cortical dysplasia D. brain tumor
(Answer: C / Reference: B 1636 - 1638 / MPL: 0.33) (Answer: C / Reference: B pp. 2007 / MPL: 0.6)
C. bed rest, steroids, surgery
34. Which of the following is TRUE regarding benign febrile seizures? D. Pen G, bed rest, paracetamol
A. Peak age of onset is 14 to 18 months (Answer: D / Reference: A p 874 879 / MPL: 0.6)
B. Associated with high incidence of epilepsy
C. Anticonvulsants is indicated to prevent recurrences 49. Which of the following is NOT a diagnostic criteria Kawasakis disease?
D. Seizures is typically focal lasting for less than 10 minutes A. Polymorphous exanthem
(Answer: A / Reference: B p 1994 / MPL: 0.9) B. Bilateral conjunctival injection with exudates
C. Spiking fever up to 40 C
35. Which of the following is the most appropriate age-independent measure of protein-calorie D. unilateral cervical lymphadenopathy
malnutrition? (Answer: B / Reference: B pp 824 - 825 / MPL: 0.6)
A. Arm span and reach
B. Height for weight ratio 50. Which of the following is a sign and/or symptom of congestive heart failure in infants?
C. Weight for age A. diaphoresis C. feeding difficulties
D. Weight for height ratio B. neck vein distention D. Bipedal edema
(Answer: D / Reference: A p 165 / MPL: 0.6) (Answer: C / Reference: B p/ 1583 / MPL: 0.6)

36. Which of the following is NOT a characteristic of Kwashiorkor? Questions 51 to 52: .A 5-year-old girl presents at the ER with gross hematuria and periorbital edema. Her
A. Dermatitis BP is 150/95mmHg.The attending physician suspects she has acute postinfectious glomerulonephritis.
B. Edema Urinalysis reveals too numerous to count RBCs and 2+ protein. To confirm the diagnosis, you measure
C. Photosensitive skin serum complement levels: C3 is 57mg/dl (low) and C4 is 24mg/dl(normal).
D. Lethargy, apathy
(Answer: C / Reference: A p 168 - 170 / MPL: 0.9) 51. What is the most common cause of postinfectious glomerulonephritis?
A. Staphylococcus aureus C. adenovirus
37. A patient with Albrights hereditary osteodystrophy (pseudohypoparathyroidism) sees his physician for B. group A beta hemolytic strep D. Streptococcus
an upper respiratory tract infection. This patient has a stocky build with a round face, short stature and epidermidis
brachydactyly of the fourth and fifth metacarpals. Which of the following is also associated with (Answer: B / Reference: A p 1743 - 1744 / MPL: 0.6)
pseudohypoparathyroidism?
A. Mental retardation 52. When do you expect the hypertension to resolve?
B. Decreased serum parathormone A. 2 weeks after onset C. 2 months after onset
C. Increased serum calcium B. 4 weeks after onset D. 1 year after onset
D. Decreased serum phosphorus (Answer: B / Reference: A p 1743 - 3744 / MPL: 0.6)
(Answer: A / Reference: B pp 1894 - 1895)
For question numbers 53 to 54. A 12-year- old boy presents to the emergency department with a 3 day
38. A 2-year-old child is brought to the emergency room for suspected physical abuse. The physician history of tea colored urine that began 2 days following an upper respiratory tract infection. There is no
performs a thorough history and physical examination to determine evidence of maltreatment. Which of history of dysuria and findings on PE are normal. The family history is negative for any kidney disease.
the following would be specific evidence of child abuse in this case? His BP is 115/70mmHg. Urinalysis reveals: sp gravity 1.025; pH 6.0; RBC too numerous to count; 1+
A. A blue, non tender macular lesion in the presacral area protein; and 0 to 2 WBC. Electrolyte levels are normal.
B. Weight less than the 5th percentile
C. Bruised knees 53. What is the most likely diagnosis?
D. A circular burn restricted to the buttocks A. Alport syndrome C. immunoglobulin A
(Answer: D / Reference: A p 1125 / MPL: 0.6) nephropathy
B. focal segmental glomerulosclerosis D. postinfectious glomerulonephritis
39. Which of the following is true regarding reporting child abuse incidents? (Answer: C / Reference: B pp 1737 - 1738 / MPL: 0.3)
A. Suspected child abuse should be confirmed before reporting
B. Only the attending physician can report a child abuse case 54. Which of the following statements is NOT TRUE of the above patient?
C. Child abuse cases should be reported within 48 hours of learning of the case A. renal function is normal C. more commonly seen in males
D. Failure to report is punishable by a fine of P500.00 and imprisonment for 1 month B. C3 is elevated D. bedrest
(Answer: C / Reference: A p 1119 / MPL: 0.6) is part of the management
(Answer: B / Reference: A p 1737 - 1738 / MPL: 0.6)
40. Which of the following would indicate Tanner Stage 3 sexual development in females?
A. Acne 55. Which of the following is true regarding laryngotracheomalacia?
B. Darkly pigmented slightly curly pubic hair A. Stridor is usually expiratory in timing
C. Fine hair on the upper lip B. Usually resolves by 18 months of age
D. Menstruation C. Secondary to RSV
(Answer: B / Reference: A p 53 - 54 / MPL: 0.6) D. Supine position will diminish the intensity of stridor
(Answer: B / Reference: A p 653 / MPL: 0.6)
For question nos. 41 to 46. The 18-month-old daughter of a famous sportsman from Baguio was rushed
to the emergency room because of acute onset of fever associated with purpura and lethargy. The 56. Which of the following is NOT TRUE regarding acute epiglottitis?
extremities were cold and pulses were faint. There was profuse bleeding coming from both nostrils. A. The most common etiologic agent is Hemophilus influenzae
Meningococcemia with DIC was the consideration. The patient is the 12 th case with meningococcemia in B. Examination of the throat may precipitate cardiorespiratory arrest
the locality. C. Associated with hyperextension of the neck and drooling of saliva
D. Steeple sign is seen on lateral neck x-rays
41. Which strain of Neisseria meningitidis is mostly likely to be responsible for the disease in the above (Answer: D / Reference: A p 1406 / MPL: 0.6)
patient?
A. Group A C. Group B 57. What is the ideal weight of an 18 month old female child?
B. Group C D Group W A. 7 kg
(Answer: C / Reference: A p 455 / MPL: 0.33) B. 9 kg
C. 11 kg
42. Which of the following is NOT TRUE regarding the above etiologic agent? D. 15 kg
A. gram negative diplococci C. obligate intracellular anaerobes (Answer: C / Reference: B pp. 31 / MPL: 0.6)
B. polysaccharide capsule D. transmitted via respiratory droplets
(Answer: C / Reference: B p 896 / MPL: 0.9) 58. Which component of DPT vaccine is highly responsible for the adverse effects after vaccination?
A. Diphtheria
43. Which of the following laboratory findings is NOT consistent with the above patient? B. Pertussis
A. thrombocytopenia C. Increased fibrinogen C. Typhoid
B. prolonged PT and PTT D. increased D-dimers D. Tetanus
(Answer: C / Reference: B pp 896 - 899 / MPL: 0.6) (Answer: B / Reference: A p 37 / MPL: 0.9)

44. Which of the following additional signs and symptoms is NOT expected to be seen in this patient? 59. Which of the following is the correct definition of a fully immunized child according to EPI?
A. Pallor C. A. At the age of 1 year, the child should receive 1 dose of BCG, 3 doses of DPT/polio/hepa B
Peripheral cyanosis and 1 dose of measles
B. Hypotension D. Capillary refill time of B. At the age of 1 year, the child should receive 1 dose of BCG, 3 doses of DPT/polio/hepa
2 seconds B/HiB and 1 dose of measles
(Answer: D / Reference: B pp 896 899 / MPL: 0.6) C. At the age of 1 year, the child should receive 1 dose of BCG, 3 doses of DPT/polio/hepa
B/HiB/Pneumococcal and 1 dose of measles
45. Which of the following should be the first management priority in this patient? D. At the age of 1 year, the child should receive 1 dose of BCG, 3 doses of DPT/polio/hepa B
A. Secure the airway C. Vascular access and A and 1 dose of measles
B. Fluid resuscitation D. Inotropes (Answer: A / Reference: A p 41 / MPL: 0.9)
(Answer: A / Reference: B p 296 / MPL: 0.6)
60. Which of the following is true regarding Rabies infection?
46. Which of the following antibiotics is NOT appropriate for the above patient? A. The incubation period ranges from 20-180 days
A. Cefotaxime C. Ceftriaxone B. Pre-exposure prophylaxis consists of a 5 dose vaccine regimen
B. Chloramphenicol D. Cefalexin C. Rabies immunoglobulin is indicated for all dog bites
(Answer: D / Reference: A p 896 - 899 / MPL: 0.6) D. Associated with 75% mortality rate

For questions 47 to 48. A 10 year old girt has a history of easy fatigability, joint pains and swelling of the Reference: B p.1102
ankles, high grade fever, systolic murmur at the apex and a positive ASO titer. MPL: 0.6

47. Which of the following is the correct diagnosis? 61. Which of the following is the most common manifestation of Candidiasis in children?
A. RHD with mitral stenosis A. Diaper dermatitis
B. Acute rheumatic fever with arthritis B. Pneumonia
C. Acute rheumatic fever with carditis and arthritis C. Urinary tract infection
D. RHD with cardiac tamponade D. Otitis media
(Answer: C / Reference: B pp. 874 - 879 / MPL: 0.6) (Answer: A / Reference: A p 1031 / MPL: 0.9)

48. Which of the following is the most appropriate management? 62. When dealing with bacterial infections, which of the following is the gold standard for diagnosis?
A. bed rest, diuretics, penicillin A. Serologic testing
B. oxygen, morphine, NSAIDs B. Latex agglutination test
C. Viral studies A. Bitots spots
D. Culture and sensitivity B. Nyctalopia
(Answer: D / Reference: B p 835 - 837 / MPL: 0.9) C. Xerophthalmia
D. Keratomalacia
63. The presence of generalized muscular spasm, trismus, opisthotonus, and laryngospasm are (Answer: B / Reference: B p. 180 / MPL: 0.9)
characteristic of which etiologic agent?
A. Treponema pallidum 78. Which of the following is NOT characteristic of the keratoconjunctivitis caused by Herpes simplex
B. Clostridium tetani virus?
C. Bordetella pertussis A. painful vesicles on the lids
D. Corynebacterium diphtheriae B. enlarged and tender preauricular nodes
(Answer: B / Reference: A p 468 - 469 / MPL: 0.9) C. keratomalacia
D. fluorescent dendritic ulcers of the cornea
64. Which of the following is the drug of choice for the above etiologic agent (No. 63)? (Answer: C / Reference: B p 1053 / MPL: 0.6)
A. Penicillin G
B. Tetracycline For question numbers 79-80. A 2 year old female was brought to the emergency room with high grade
C. Amoxycillin fever, and diffuse rash (onset of 5 days ago), and a swollen right hand. The rashes were noted to be
D. Erythromycin vesiculopustular over the trunk, face and extremities. Crusting were noted in some of the lesions. The
(Answer: A / Reference: A p 469; B p 952 / MPL: 0.9) right hand was noted to swollen, erythematous and tender. CBC showed leukocytosis.

65. A 7-year-old girl is brought to you for chronic cough, low grade fever and mild weight loss for more 79. Which of the following would be the most appropriate therapeutic measures for the patient?
than a month. Which of the following is your primary consideration? A. Give oral acyclovir
A. Streptococcus pneumoniae infection B. Give intravenous cephalexin
B. Pertussis C. Give intravenous acyclovir
C. Mycobacterium tuberculosis infection D. Paracetamol only
D. Influenza (Answer: B / Reference: B p. 1061 / MPL: 0.9)
(Answer: C / Reference: A p 503 / MPL: 0.9)
C80. Which of the following is the main mode of transmission of the causative agent for the diffuse rashes
66. Which of the following is NOT a common clinical manifestation of Infection with Hemophilius of the patient?
influenzae? A. Skin to lesion contact
A. Meningitis B. Oral-fecal route
B. Septic arthritis C. Respiratory route
C. Urinary tract infections D. Local inoculation
D. Epiglottitis (Answer: C / Reference: B p 1057 / MPL: 0.9)
(Answer: C / Reference: B pp 905 - 907 / MPL: 0.6)
81. The most characteristic sign of rubella:
67. Which of the following is characteristic of roseola iinfantum? maculopapular rash on the trunk and extremities
A. Macular rash after fever of 3 days duration Discrete rose spots in the soft palate
B. High grad fever and rashes occurring at the onset of fever Retroauricular, posterior cervical and postoccipital lymphadenopathy
C. Lymphadenopathy, low grade fever Branny desquamation
D. Vesiculopustular rashes after defervescence (Answer: C / Reference: B p. 1032 / MPL: 0.9)
(Answer: A / Reference: B p. 1070 / MPL: 0.9)
82.. The cardiac structural defects associated with congenital rubella syndrome:
68. Which of the following is NOT part of the management of children with Rubeola? A. patent ductus arteriosus and ventricular septal defect
A. Antistaphylococcal antibiotics for bronchopneumonia B. atrial septal defect and ventricular septal defect
B. Vitamin A supplementation C. ventricular septal defect and pulmonary artery stenosis
C. Isolation until 4 days after the appearance of rash D. pulmonary artery stenosis and patent ductus arteriosus
D. Give Measles vaccination on recovery (Answer: D / Reference: B p 1033 / MPL: 0.6)
(Answer: D / Reference: B p 1029 / MPL: 0.9)
83. Which of the following conditions does not present with fetal infection?
69. Which of the following is NOT true regarding mumps? A. Rubella
A. Commonly affects the parotid gland B. Rubeola
B. Infectious up to 5 days after onset of swelling C.Parvovirus B19 infection
C. 2 doses of MMR vaccine confers full protection D. Varicella Zoster virus infection
D. All males infected with mumps will develop orchitis (Answer: B / Reference: B p. 1033, 1050, 1059 / MPL: 0.6)
(Answer: D / Reference: B p. 1036 / MPL: 0.6)
84. The highest incidence of diarrhea occurs in this age group:
70. Which of the following is TRUE regarding Poliovirus infection? A. 0-5 months C. 12-17 months
A. OPV administration results in fecal viral excretion for a few days B. 6-11 months D. 18-24 months
B. OPV can result in vaccine associated paralytic polio (Answer: B / Reference: D / MPL: 0.9)
C. Mass vaccination requires the use of IPV
D. IPV is more potent than OPV 85. The pathogenic mechanism involved in rotavirus diarrhea is:
(Answer: B / Reference: B p. 1042 / MPL: 0.6) A. loss of monosaccharidases
B. mucosal adhesion and invasion
71. Rubella vaccination is contraindicated in all of the following, EXCEPT? C. patchy destruction of the villous epithelium
A. Pregnant women D. production of toxin which increases secretory function
B. Women of child-bearing age (Answer: C / Reference: D / MPL: 0.9)
C. Patients on prednisone
D. Patients given chemotherapy 86. The drug of choice for cholera:
(Answer: B / Reference: B p 1033 / MPL: 0.9) A. Cotrimoxazole C. Furazolidone
B. Tetracycline D. Metronidazole
72. A 6 month old infant came in for 4 days moderate grade fever, dry hacking cough and coryza. Koplik (Answer: B / Reference: D / MPL: 0.9)
spots were seen on examination of the oral cavity. Which part of the oral cavity are the Koplik spots best
appreciated? 87. The first teeth to erupt are usually the:
A. opposite the lower canines A. central incisors C. first molars
B. opposite the lower molars B. lateral incisors D. second molars
C. palate (Answer: A / Reference: A p 60 / MPL: 0.9)
D. peritonsillar areas
(Answer: B / Reference: B p 1027 / MPL: 0.9) 88. Which of the following senses is most developed in newborns?
A. auditory C. taste
73. Lifetime immunity is acquired by recovery from the following viral infection EXCEPT: B. tactile D. visual
A. Mumps C. Rubella (Answer: B / Reference: A p 53 - 54 / MPL: 0.6)
B. Dengue fever D. Varicella
(Answer: B / Reference: B p 1033, 1036, 1061 / MPL: 0.9) 89. Visual acuity of 20/20 is achieved by:
A. 2 yrs old C. 5 yrs old
74. A 3-year-old female child who has been receiving chemotherapy for acute lymphocytic leukemia was B. 4 yrs old D. 10 yrs old
exposed to her sibling with chicken pox. Mother claimed she was not given immunization. Which (Answer: C / Reference: B p. 2084 / MPL: 0.6)
prophylactic measure should be administered to this child?
A. Varicella vaccine immediately 90. A 3 year old can perform all of the following, EXCEPT:
B. Varicella zoster immunoglobulin immediately A. draws man with 3 parts
C. Oral acyclovir B. rides a tricycle
D. None, just isolate the child from the sibling C. tells little stories about experiences
(Answer: B / Reference: A p 1061 - 1062 / MPL: 0.6) D. helps in simple house task
(Answer: A / Reference: A p 112 / MPL: 0.6)
75. Which of the following is NOT true regarding varicella vaccine?
A. it should be given intramuscularly 91. A 15 month old is able to do all of the following, EXCEPT:
B. may be given simultaneously with other vaccines A. seats self in childs chair
C. Contains a live attenuated strain of varicella virus B. pats pictures
D. 2 doses are required for children < 12 years old C. attempts to use a spoon
(Answer: D / Reference: C p 680 - 681 / MPL: 0.6) D. imitates a vertical stroke
(Answer: A / Reference: A p 111 / MPL: 0.6)
76. Which of the following is considered the infectious period of varicella?
A. 1 day before and 5-7 days after the onset of rash 92. What is the best stimulus to breastmilk secretion in lactating mothers?
B. 1 week before and 2 weeks after the rash A. plenty of fluids and vitamins
C. during the appearance of the rash and 2 weeks after B. good rest and high caloric intake
D. during the prodrome period and 10 days after the disappearance of the rash C. eat more shellfish such as clams and tahong
(Answer: A / Reference: B p 1057 / MPL: 0.6) D. frequent sucking and complete emptying of breast
(Answer: D / Reference: A p 142 / MPL: 0.9)
77. Which of the following is not an eye sign of avitaminosis A ?
93. Irritability, pruritus, painful extremities with brawny swelling, coarse hair, dry skin, seborrhea and C - All nutrient needs of infants must be met by 6 months. But during this time breast milk
increased intracranial pressure is seen in: volume and iron stores may not be adequate to accommodate the demands of growth Iron deficiency
A. Hypervitaminosis A C. Hypovitaminosis D anemia may ensue. (Nelsons Textbook of Pediatrics, 17th ed, 2004, p. 164)
B. Hypervitaminosis D D. Hypocalcemia
(Answer: A / Reference: A p 1135 / MPL: 0.6) 4. Chronic intake of carotenoids may result in:
A. Pseudotumor cerebri
94. A 7-year- old presents with the following manifestations: skin and mucous membrane lesions, B. Yellow skin and sclerae
conjunctivitis, glossitis, depression and somnolence. What is the most likely deficiency? C. Yellow skin
A. Vitamin B1 C. Vitamin D. Cranial nerve palsy
B12 (Recall) - GIT
B. Vitamin B6 D. niacin C - Carotene (yellow pigment); carotenemia although non-toxic is due to deposition of
(Answer: B / Reference: B p. 183 / MPL: 0.6) carotene pigments in the skin but not the sclerae, (B) Icteresia and jaundice is yellowish discoloration of
the sclerae and skin secondary to deposition of bilirubin due to a pathology in Bilirubin metabolism
95. Which of the following can be considered as a (+) Mantoux test? hemolytic of hepatobiliary disease (A and D) Pseudotumor cerebri and cranial nerve palsy are CNS
A. A 2 year old boy, no BCG, Mantoux 4mm induration manifestations of Vitamin Atoxicity after chronic intake of >100,000/u/day vitamin A.
B. A 4 year old girl, (+) BCG, Mantoux 8mm induration (Nelsons Textbook of Pediatrics, 17th ed, 2004, p. 181)
C. A 2 month old male infant, (+) BCG, Mantoux 3mm induration
D. A 4 year old boy, (+) BCG, Mantoux 12mm induration 5. A 3-hour old newborn with a prenatal history of maternal hydramnios was noted to have
(Answer: D / Reference: A p 505 / MPL: 0.33) frothing of mouth and nose with circumoral cyanosis. You anticipate that:
A. There is inability to pass the nasogastric tube
96. Which laboratory tests are helpful in establishing the etiology of anemia? B. Presence of scaphoid abdomen
A. hemoglobin, reticulocyte count, bone marrow exam C. Referral to ENT will be done
B. serum iron, CBC, platelet count D. All of the above
C. CBC, serum iron, reticulocyte count (Problem Solving) - GIT
D. CBC, smear review, reticulocyte count, iron studies A - In early onset respiratory distress, inability to pass an NGT suggests esophageal atresia
(Answer: D / Reference: A p 1065 / MPL: 0.6) with TEF. This is a surgical problem not seen by ENT (C) maternal polyhydramnios is more associated
with TEF rather than diaphragmatic hernia (B) (Nelsons Textbook of Pediatrics, 17 th ed, 2004,Chapter
97. Which of the following is NOT a feature of hereditary spherocytosis? 300)
A. >20% spherocytes
B. sex linked recessive 6. Which of the micronutrients does not have recognized anti-infective properties?
C. splenomegaly A. Vitamin A
D. cholelithiasis B. Vitamin D
(Answer: B / Reference: A p 1620 - 1621125 / MPL: 0.6) C. Iron
D. Zinc
98. Which of the following is NOT a feature of aplastic anemia? (Recall) - GIT
A. leucopenia B - Deficiency of any essential nutrient may result in failure to thrive and accompanying lack
B. thrombocytopenia of immune protection. However, infections are more common in children with Vitamin. A, Iron,
C. hypoferritinemia Zincdeficiencies. These 3 have roles in the immune system. (Nelsons Textbook of Pediatrics, 17 th ed,
D. neutropenia 2004, Chapter 44)
(Answer: C / Reference: A p 1288 1289; MPL: 0.9)
7. A 1-1/2 year old is discovered to have a bottle of alkali solution in his mouth. The bottle was
99. Which of the following is NOT a favorable prognostic factor of acute lymphoblastic leukemia? noted to be half empty. No external signs on the child's face were seen. Your advice is
A. 5 years old C. to bring the child to the ER. There must be prior administration of:
Hemoglobin: 11 gms A. Emetic
B. female D. absence of CNS B. Antiemetic
leukemia C. Milk
(Answer: C / Reference: A p 1694 - 1696 D. Laxative
(Problem Solving) - GIT
100. The most important index for protein malnutrition at present is: C - Milk calms the child and dilutes the alkali (A) dont induce emesis (Nelsons Textbook
A. edema C. negative nitrogen of Pediatrics, 17th ed, 2004, (Chapter 308.2)
balance
B. wasting D. slow growth and 8. A 3 year old accidentally ingested a coin. A chest x-ray was taken. In contrast to foreign body
development trachea, the coin in the esophagus as seen on radiograph will show:
(Answer: A / Reference: A p 168 - 169 / MPL: 0.6) A. Edge of the coin in AP view
B. Edge of coin on lateral view
END OF EXAMINATION C. Flat surface in AP view
D. A and C
Legend for the references: (Problem Solving) - GIT
A Textbook of Pediatrics and Child Health by del Mundo et al 4th Edition D - (Nelsons Textbook of Pediatrics, 17th ed, 2004, Chapter 308.1)
B Nelsons Textbook of Pediatrics by Behrman 17th Edition
C Red Book 26th Edition, 2003 Report of the Committee of Infectious Disease (AAP) 9. The WHO recommends the use of ORS in developing countries to have a sodium concentration
D Readings on Diarrhea of _____ mmol/L:
A. 90
B. 100
C. 110
D. 120
(Recall) - GIT
University of the East A - 90 mmol/l
RAMON MAGSAYSAY MEMORIAL MEDICAL CENTER Above 90 is hyperosmolar (Nelsons Textbook of Pediatrics, 17 th ed, 2004, p. 250)
FEBRUARY 2005
10. A 3-week old with essentially normal birth history had episodes of intermittent vomiting after
feeding.
CHOOSE THE BEST ANSWER: If pyloric stenosis is being considered, you expect the following EXCEPT:
A. Hypochloremic alkalosis
1 A newborn FT was noted to be pale at 4th month of life. Iron is unlikely in this condition because B. Bilous vomiting
infants have sufficient stores to meet their iron requirement for: C. Gastric peristaltic wave
A. 2-3 months D. Olive-shaped RUQ mass in abdominal palpation
B. 46 months (Problem Solving) - GIT
C. 7-8 months C - The hallmark of gastric obstruction is non-bilious vomiting.(Nelsons Textbook of
th
D. 10-12 months Pediatrics, 17 ed, 2004, Chapter 310)
(Problem Solving) - GIT
B It is by 6 months that iron should be supplemented among healthy full terms. 11. A one day old had bilous vomiting. He was noted to be slightly jaundiced. The abdomen was
(Nelsons Textbook of Pediatrics, 17th ed, 2004, p. 156) not distended but there was occasional visible peristaltic nerves on the abdominal wall. Plain
abdomen x-ray showed double-bubble sign. The obstruction is on what level?
2 The breastfed baby of a pure vegetarian mother may develop: A. Distal esophagus
A. Xerophthalmia B. Gastric
B. Diarrhea & Dementia C. Duodenal
C. Osteomalacia D. Colonic
D. Anemia (Problem Solving) - GIT
(Problem Solving) - GIT C - (Nelsons Textbook of Pediatrics, 17th ed, 2004, p. 1233)
D - Strict Vegan diets contain no eggs, meat or milk products making this deficient in When the obstruction is in the duodenum beyond the Ampula of Vater vomitus is
Vitamin B 12. bilous. The Ampulla of Vater is the site where bile exits.
(Chap. 446 p. 1612)
Nursing Vegan mothers must be given B12 to prevent. Methylmalonic academia and 12. A 2-year old with head trauma underwent a neurosurgical procedure. At the PICU he had
anemia in their infants. (166) (A) Xerophthalmia is Vitamin A deficiency and for which Vitamin A rich massive hematemesis. You would consider:
sources are the vegetables. (B) Diarrhea and dementia are signs of niacin deficiency (Pellagra) B A. Curlings ulcer
vitamins come from grains and vegetables. (C) Osteomalacia is Vitamin D deficiency (Rickets) B. H. pylori infection
(Nelsons Textbook of Pediatrics, 17th ed, 2004, Chap. 446 Table 44-I) C. Cushings ulcer
D. B and C
3 By 6th month of age micronutrients must be started. Foremost among these is that nutrient (Problem Solving) - GIT
that prevents: C - (Nelsons Textbook of Pediatrics, 17th ed, 2004, Chapter 316.1)
A. Xerophthalmia Gastric hypersecretion is associated with head trauma and severe CNS disorders
B. Scurvy (A) Curlings ulcers are associated with severe burns
C. Anemia (B) The course is too acute for H. pylori infection
D. Goiter
(Problem Solving) - GIT 13. An 11 year old Tanner stage 2 female developed epigastric pain / 8 hours later there was fever
nausea and vomiting. She passed 2 soft bowel movements. In the clinic, she limps and
abdominal palpation, there was generalized guarding. Most likely, she has:
A. Pelvic inflammatory disease A - Neuroblastoma is an embryonal cancer of the peripheral sympathetic nervous system. It
B. Ruptured ectopic pregnancy is the third most common pediatric cancer accounting for about 8% of pediatric cases. It is the most
C. Appendicitis common malignancy in infancy accounting for 28-39% of neonatal malignancies. Mixed embryonal
D. Mesenteric adenitis neoplasm composing of three elements (Choice B) pertains to Wilms tumor. Leukoria (Choice C) is the
(Problem Solving) - GIT characteristic clinical presentation of retinoblastoma. Diagnosis does not require a biopsy (Choice D) in
C - All choices are differentials of appendicitis retinoblastoma since characteristic ophthalmologic findings are sufficient. (Nelsons Textbook of
(A) PID presents with vaginal discharge Pediatrics, 17th ed, 2004, Chap. 490)
(B) Pregnancy is unlikely for Tanner 2 22. Among the following tumors, the one with the best over-all survival rate is:
(D) Mesenteric adenitis follows a week of respiratory infection
(Nelsons Textbook of Pediatrics, 17th ed, 2004, Chapter 324) A. Wilms tumor
B. Non-Hodgkins lymphoma
14. A 2 year old previously well child had intermittent crying episodes and projectile vomiting 12 C. Hepatoblastoma
hours ago. There was gassy abdominal distention and passage of maroon-colored stools. You D. Neuroblastoma
would: (Problem Solving) Hema/Onco
A. Give antiamebics and antiemetics A - Prognosis of neuroblastoma is generally good. Survival in low risk group is 91-100%;
B. Do abdominal x-ray and refer to surgery average group 75-98% Stage 4S carries 100% survival with supportive care only because the tumor
C. Give antibiotics regresses spontaneously. Wilms tumor (Choice A) prognostic factors are tumor size, stage and
D. All of the above histology. More than 60% of patients with all stages generally survive. Non-Hodgkins lymphoma (Choice
(Problem Solving) - GIT B) is considered disseminated disease from the time of diagnosis. Hepatoblastoma (Choice C) if
B - The diagnosis is intussusception unresected carries survival rate of 60%.
(A) and (C) are not employed in intussusception (Nelsons Textbook of Pediatrics, 17th ed, 2004, Chap. 491)
(Nelson Textbook of Pediatrics, 17th ed, 2004)
23. Which of the following patterns noted on continuous monitoring of fetal heart rate is most
15. The GI malignancy prevented by immunization is: indicative of fetal distress?
A. Gastric carcinoma A. Baseline variability with periodic acceleration
B. Colonic carcinoma B. Increasing baseline variability
C. Hepatic carcinoma C. Early deceleration without baseline variability
D. Pancreatic carcinoma D. Late deceleration without baseline variability
(Recall) - GIT (Problem Solving) - Neonatology
C - Hepatitis B directly increases the risk of Hepatocarcinoma in later life. This is prevented D - Baseline variability with or without periodic acceleration of the heart rate is a sign of fetal
by vaccination. There are no known vaccines for preventing the onset of the other cancers. Gastric well-being. Increasing baseline variability may represent early compromise of fetal oxygenation. The
cancer from H. pylori gastritis is well documented. So far there are no H. pylori vaccines. early deceleration pattern is due to pressure of the anterior fontanelle on the cervix and is not a sign of
(Nelsons Textbook of Pediatrics, 17th ed, 2004, p. 1328) fetal distress. The variable deceleration pattern indicates umbilical cord compression. The late
deceleration pattern signifies fetal hypoxemia. (Behrman, ed. 13, p. 368)
16. Prolonged antibiotic therapy can result to bleeding with the following laboratory results:
A. Normal PT, normal PTT 24. A healthy premature infant who weighs 950 g (2 lb, 1 1/2 oz) is fed undiluted breast milk to
B. Prolonged PT, prolonged PTT provide 120 cal/kg per day. Over ensuing weeks the baby is most apt to develop:
C. Prolonged PT, normal PTT A. Hypernatremia
D. Normal PT, prolonged PTT B. Hypocalcemia
(Problem Solving) Hema/Onco C. Blood in the stool
C - Prolonged antibiotic therapy can lead to gut sterilization leading to reduced synthesis of D. Metabolic acidosis
Vitamin K dependent clotting factors (Factors II, VII, IX & X, protein C and protein S). This reduction of (Problem Solving) - Neonatology
clotting factors of the extrinsic limb of coagulation will lead to prolongation of Prothrombin Time with B - Breast milk has much less calcium and phosphorus than do commercial formulas.
normal PTT. (Behrman, ed. 113, pp. 162-163)
(Nelsons Textbook of Pediatrics, 17th ed, 2004)
17. The CBC of a 7-yer old male with epistaxis and ecchymoses revealed Hgb 67 g/dl, Hct 18%, 25. An infant weighing 1400 g (3 lb) is born at 32 weeks gestation in a delivery room that has an
WBC 50,000, Neutrophils 5%, Lymphoblast 95%, Platelet Count 20,000. What is you primary ambient temperature of 24'C. Within a few minutes of birth, this infant is likely to exhibit all the
consideration? following EXCEPT:
A. Aplastic anemia A. Pallor
B. Acute lymphocytic leukemia B. Shivering
C. Disseminated intravascular coagulation C. A fall in body temperature
D. Idiopathic thrombocytopenic purpura D. Increased respiratory rate
(Problem Solving) Hema/Onco (Problem Solving) - Neonatology
B - The anemia and thrombocytopenia are due to decreased production of erythroid and B - A room temperature of 24C provides a cold environment for preterm infants weighing
megakaryocytic precursors resulting from blastic proliferation in the bone marrow. Aplastic anemia less than 1500 g. Aside from the fact that these infants emerge from a warm intrauterine environment. In
(Choice a) is associated with pancytopenia. DIC (Choice C) doesnt produce leukocytosis and blasts in order to bring body temperature back to normal they must increase their metabolic rate; ventilation in
the peripheral smear. It is associated with the coagulation mechanism, not the hematopoietic cells. ITP turn, must increase proportionally to ensure adequate oxygen supply. Infants rarely shiver in response to
(Choice D) is only associated with thrombocytopenia. No leukocytosis and blasts are seen in the a need to increase heat production. (Behrman, ed. 113, p. 363)
peripheral smear. (Nelsons Textbook of Pediatrics, 17th ed, 2004)
26. Initial examination of a full tem infant weighing less than 2500 g (5 lb, 8 oz) shows edema over
18. Recurrent gum bleeding was noted in a 7-year old female. CBC and platelet count are normal, the dorsum of her hands and feet. Which of the following findings would support a diagnosis of
Prothrombin time is normal but bleeding time and partial thromboplastin time are prolonged. The Turner's syndrome?
most likely diagnosis is: A. A liver palpable to 2 cm below the costal margin
A. ITP B. Tremulous movements and ankle clonus
B. Hypoprothrombinemia C. Redundant skin folds at the nape of the neck
C. TTP D. A transient, longitudinal division of the body into a red half and a pale
D. Von Willebrand Disease half
(Problem Solving) Hema/Onco (Problem Solving) - Neonatology
D - Von Willebrand disease is a disorder associated with mucocutaneous hemorrhages. C - Turners syndrome is a genetic disorder with the 45XO karyotype being most common.
The disorder is due to deficiency of Von Willebrand factor, a glycoprotein that is synthesized in At birth affected infants have low weights, short stature, edema over the dorsum of hands and feet and
megakaryocytes and endothelial cells. During normal hemostasis VWF adheres to the endothelial matrix loose skin folds at the nape of the neck. (Behrman, ed. 13, pp. 264-266. 1236-1237)
after vascular damage. Changes in the conformation of VWF cause platelets to be an adhere to
VWF resulting to platelet activation and recruitment of additional platelets. VWF also serves as the 27. Object permanence is not present in a 2 months old, whose response to dropping a ball is:
carrier protein for plasma factor VIII. Severe deficiency of VWF can cause prolongation of bleeding A. Staring descending as the ball descends
time and PTT. ITP (Choice A) results only to prolonged BT because the coagulation phase is not B. Eyes descending as the ball hits the ground
affected. Hypoprothrombinemia (Choice B) results to decreased synthesis of Vitamin K dependent C. Crying when the ball hits the ground
factor causing prolonged PT. ITP (Choice C) is a form of microangiopathic hemolytic anemia with D. Smiling at the game of the hide-and-seek
thrombocytopenia. (Nelsons Textbook of Pediatrics, 17th ed, 2004, Chap. 469)
(Problem Solving) - Neonatology
19. A 10-kg child with iron deficiency anemia should receive: A - Out of sight out of mind is the characteristic response of a 2 month old. Object
A. 20 mg elemental iron permanence appears at approximately 8 months of age. This is also called object constancy.
B. 30 mg elemental iron (Nelson Textbook of Pediatrics, 15th ed, Chap. 11))
C. 60 mg elemental iron
D. 70 mg elemental iron 28. The ability to manipulate small objects with the pincer grasp is usually noted at what age?
(Recall) Hema/Onco A. 0 to 2 months
C - The therapeutic dose of elemental iron is 6 mkd. B. 3 to 5 months
(Nelsons Textbook of Pediatrics, 17th ed, 2004, Chap. 447) C. 6 to 7 months
D. 8 to 9 months
20. The following statement is true regarding brain tumors in childhood: (Recall) - Neonatology
A. Hereditary syndromes are associated with increased incidence of brain tumors in 25%of cases D - The pincer grasp, which is noted at age 8 to 9 months, along with increasing mobility,
B. Cranial exposure to ionizing radiation is associated with increased incidence of brain tumor enables an infant to explore the environment. (Nelson Textbook of Pediatrics, 17 th ed, 2004)
C. Supratentorial tumors predominate among children aged 1-10 years
D. In general, there is a slight predominance of supratentorial tumor location in children
(Recall) Hema/Onco 29. A developmentally normal child who is able to run, build a tower of two cubes, pretend play
B - Cranial exposure to ionizing radiation has been shown to be associated with increased with a doll and speak in two-word sentences is what age?
incidence of brain tumors. This has been observed in pediatric acute lymphocytic leukemia who A. 19 months
underwent craniospinal prophylaxis. Hereditary syndrome (Choice A) are associated with increased B. 15 months
incidence only in 5% of cases. Supratentorial tumors predominate during the 1 st year of life thus Choice C. 14 months
C is incorrect. Generally, infratentorial tumors predominate slightly over supratentorial tumors making D. 24 months
Choice D incorrect (Nelsons Textbook of Pediatrics, 17th ed, 2004, Chap. 489 (Problem Solving) - Neonatology
A - (See Table 11-3, Chapter 11, Nelson Textbook of Pediatrics, 15th ed)
21. Neuroblastoma is a condition characterized by the following:
A. Malignancy most frequently diagnosed in infancy 30. A developmentally normal child who is just able to sit without support, transferobjects from
B. Mixed embryonal neoplasm composed of three elements: blastoma, epithelia and stroma hand to hand, and speak in a monosyllabic babble is probably what age?
C. Classically presents with leukocoria A. 2 months
D. Diagnosis does not require a biopsy but is established characteristical clinical findings B. 4 months
(Problem Solving) Hema/Onco C. 9 months
D. 6 months
(Problem Solving) - Neonatology C. Osteochondroma
D - (Nelson Textbook of Pediatrics, 15th ed, See Table 11-3, Chap 11) D. Osterblastoma
(Recall) Musculoskeletal Disorders
31. This primitive reflex is observed in a normal one year old: B - Osteosarcoma is the most common primary malignant bone tumor in children and
A. Tonic neck reflex adolescents, followed by Ewing sarcoma. In children younger than 10 years of age, Ewing sarcoma is
B. Parachute reflex more common than osteosarcoma. Both tumor types occur most frequently in the 2nd decade of life.
C. Palmar grasp (Ref. Nelsons Textbook of Pediatrics, 17th ed., 2004, pp. 1717)
D. Placing reflex
(Recall) - Neurology 39. An adolescent male basketball enthusiast consults you with a painful bump below his right
B - Among these 4 choices, it is B that persists normally beyond the neonatal period. In knee. He denies fever or trauma. Which of the following is the most likely diagnosis?
fact, the parachute reflex persists for life. (Nelsons Textbook of Pediatrics, 17 th ed, 2004, p. 1978) A. Legg-Calve Perthes Disease
B. Osteoid osteoma
32. Which of the following case scenarios merit an EEG as an initial test as part of the C. Osgood-Schlatter disease
neurodiagnostic evaluation? D. Osteomyelitis
A. Febrile seizure (Problem Solving) Musculoskeletal Disorders
B. First non-febrile seizure C Osgood-Schlatter disease occurs in active children, particularly during late childhood or
C. Meningitis adolescence, especially in athletes, and consists of the tearing of cartilage from the tibial tuberosity by the
D. Intracranial SOL ligamentum patellae. The child presents with pain and swelling at the site of one or both tibial tubercles.
(Problem Solving) - Neurology Rest, restriction of activities, and occasionally, a knee immobilizer may be necessary combined with
B - While the first febrile seizure is generally a benign one, an EEG is requested if it recurs. isometric exercise program. Complete resolution of symptoms through physiologic healing (physeal
The EEG provides characterization of seizure types which allows for the specific medical or surgical closure) of the tibia tubercle usually requires 12-24 months. (Nelsons Textbook of Pediatrics, 17 th ed.,
management. A lumbar puncture with CSF analysis would have confirmed meningitis. A brain CT scan 2004, pp. 2272).
would have demonstrated the intracranial SOL.(Nelsons Textbook of Pediatrics, 17 th ed. ; Behrman,
Kliegman & Jenson, 2004, p. 1978) 40. An overweight adolescent male complains of pain in the medial aspect of his knee. He denies
trauma, and he has not had a fever. The most likely diagnosis is:
33. A 2-year old boy was admitted because of low to moderate grade fever of 3 weeks, on and off A. Toxic synovitis
frontal headache of 1 week, squinting of 1 day, one episode of generalized seizure of 2 minute B. Legg-Calve-Perthes disease
duration 6 hours prior to admission. No medical consult done. No medications given except C. Medial collateral ligament strain
paracetamol. Which of the following clinical consideration is NOT COMPATIBLE with this D. Slipped capital femoral epiphysis
history? (Problem Solving) Musculoskeletal Disorders
A. Acute meningococcal meningitis D - Slipped capital femoral epiphysis (SCFE) is the most common adolescent hip disorder
B. TB meningitis with an unknown cause, in which there is a displacement of the femoral head from the femoral neck prior
C. Cryptococcal meningitis to epiphyseal closure. Common in obese adolescent boys, it presents with pain, limp, or refusal to
D. Brain abscess of otogenic origin walk. The pain may be referred to the knee or thigh. Legg-Calve-Perthes Disease, avascular necrosis
(Problem Solving) - Neurology of the femoral head presents with joint stiffness, hip and pain in the hip, thigh, knee, or groin of several
A - Except for A, all the rest are compatible of the history, presenting with the clinical weeks to months. Boys between 1-12 years (average 7 years) are most commonly affected. Toxic
manifestations of at least 2 weeks. (Nelsons Textbook of Pediatrics, 17th ed., 2004, pp. 965 ; 2040-44) synovitis is a transient inflammatory arthritis of the hip associated with fever.
(Nelsons Textbook of Pediatrics, 17th ed., 2004, pp. 2276-2279)
34. A mother calls to inform you that her previously well 4-year old child has been complaining of
headaches for about a month. For the past two weeks he has been keeping his hand in a tilted 41. This statement is NOT true about infective endocarditis:
position, and for the past few days he has been vomiting in the morning. The most likely A. In 90% of cases, the causative agent is recovered from the first 2 blood cultures
diagnosis is: B. Timing of phlebotomy is important because bacteremia occurs only during the febrile state
A. Meningitis C. Antimicrobial pretreatment of the patients reduces the yield of blood cultures to 50-60%
B. Degeneration brain disease D. Laboratory should be notified that endocarditis is suspected so that the blood can be
C. Brain abscess cultures on enriched media for more than 7 days
D. Brain tumor (Problem Solving) - Cardiovascular
(Problem Solving) - Neurology B - Timing of collection is not important because bacteremia can be expected to be relatively constant.
D - Frequently, meningitis or CNS infections will present with fever, headache, and signs of (A) It is true that in 90% of cases, the causative agent is recovered from the first 2 blood collection
irritability. Brain abscess, because it behaves like an intracranial SOL, will present as low grade fever, (C ) It is true that pretreatment with antimicrobials of the patients with bacterial endocarditis reduces
headache, and localizing signs. The hallmark of neurodegenerative disease is progressive deterioration the yield of blood culture to 50-60% (D) It is true that the laboratory should be notified that endocarditis
of neurologic functions with loss of speech, vision, hearing, or locomotion, often associated with is suspected so that if necessary the blood can be cultured on enriched media for longer than 7 days to
seizures, feeding difficulties, and impairment of intellect. Generally, brain tumors present with signs and detect nutritionally deficient and fastidious bacteria or fungi. And laboratory should be notified that the
symptoms relating to increased intracranial pressure (vomiting, lethargy, irritability) and focal neurologic patient has received antibiotics so that more sophisticated methods can be used to recover the offending
deficits. Within the 1st year of life, supratentorial tumors predominate and include, most commonly, organisms. (Nelsons Textbook of Pediatrics, 17th ed, 2004)
choroids plexus complex tumors and teratomas.. From 1-10 years of age, infratentorial tumors
predominate, owing to the high incidence of juvenile pilocytic astrocytoma and medulloblastoma. After 10 42. Painless small erythematous or hemorrhagic lesion on the palms and soles are classic lesion
years of age, supratentorial tumors again predominate, with the diffuse astrocytomas, most common. in:
(Nelsons Textbook of Pediatrics, 17th ed, 2004, pp. 1703, 2029, 2038, 2047) A. Osler nodic
B. Janeway lesions
35. Clinical evidence backs up the use of IV dexamethasone as an adjunctive therapy in C. Roth spots
acute meningitis caused by _____: D. Spincter Hemorrhages
A. Neisseria meningitidis
B. Streptococcus pneumoniae
C. Hemophilia influenza (Recall) - Cardiovascular
D. Listeria monocytogenes B - Janeway lesion are painless small erythematous or hemorrhagic lesions on the palms and soles. (A)
(Recall) - Neurology Osler nodes are tender pea-sized intradermal nodule in the pads of the fingers and toes. These lesions
C - Data support the use of IV dexamethasone, 0.15 mg/kg/dose given every 6 hours x 2 may represent vasculitis produced by circulating antigen antibody complexes (C ) Sphincter hemorrhages
days with bacterial meningitis caused by Hemophilus influenzae type b, but not with other bacterial are linear lesions beneath the nodes (D) Roth spots immune complex phenomena and seen in the
causes, in terms of less fever, lower CSF protein and lactate levels, and a reduction in permanent eyes (Nelsons Textbook of Pediatrics, 17th ed, 2004)
auditory nerve damage, as manifested by sensoneural loss.
(Nelsons Textbook of Pediatrics, 17th ed, 2004, p. 2043)
43. Neonatal circulation is NOT characterized by:
36. A 12-year old child is admitted because of the sudden onset of coma. The child had been well A. In the presence of cardiopulmonary disease PDA may remain patent
until about 6 hours prior to admission, when he began to complain of a headache. The headache B. Foramen ovale may persistently be functional
became more severe, and the child lapsed into coma. Physical examination: T = 38.2'C, flaccid C. The wall thickens and muscle mass of the neonatal (L) and (R )
and comatose. CSF: bloody: after centrifugation, the fluid appears xanthochromic, RBC = 3,000, ventricles are almost equal
WBC 7/mm3 , protein 400 mg/dl, glucose is 62 mg/dl. The most likely etiology of the coma is: D. The pulmonary vasculature is insensitive to changed pO4 and PC02
A. Intraventricular hemorrhage levels an acidosis
B. Subarachnoid hemorrhage (Problem Solving) - Cardiovascular
C. Viral encephalitis D - the pulmonary vasculature is very reactive to changes in pCO2, pO2 and pH by
D. Subdural effusion vigorous vascular constriction (A) in the presence of cardiopulmonary disease resulting to hypoxemia
(Problem Solving) - Neurology may cause the PDA to remain open. Normal PDA functionally closes by the 10-15th hour of life
B - The event is something acute, dramatic, catastrophic so the choices would only be (B) Foramen ovale is functionally closed by the 3rd months of life (C ) the wall thickness and muscle mass
between A and B. Intracranial bleeding may occur in the subarachnoid space or the bleeding may be of the ventricles right and left are almost equal. Without the placenta, and the closure of the ductus
primarily located in the parenchyma of the brain. Subarachnoid bleeding characterized by severe venosus, the left ventricle is now coupled to the high resistance systemic circulation whereas the right
headache, nuchal rigidity, and progressive low of consciousness, and intracerebral bleeding is a common ventricle is now coupled with the low resistance pulmonary circulation and the wall is slightly thickened as
event in premature infant (intraventricular hemorrhage). Rupture of an arteriovenous malformation (AV wel (Nelsons Textbook of Pediatrics, 17th ed, 2004)
mal) may occur at any age, and causes severe headache, vomiting, nuchal rigidity caused by
subarachnoid bleeding, progressive hemiparesis and a focal or generalized seizure. 44. The clinical manifestation of large VSD in neonatal patients does not include:
(Nelsons Textbook of Pediatrics, 17th ed, 2004, pp. 2036, 562) A. Systolic murmur may not be audible
B. Dyspnea
37. The metaphyseal ends of long bones are common sites of osteomyelitis. This condition C. Profuse perspiration
occurs because: D. Recurrent pulmonary infection
A. Relative anoxia promotes bacterial growth (Problem Solving) - Cardiovascular
B. There is blood pooling and reduced phagocytic activity A - systolic murmur may not be audible this occurs only in small VSD this is due to the fact
C. They are closer to the skin surface that the left to right shunt may be minimal because of the higher right sided pressure(B) Dyspnea
D. They are common sites of trauma happens because of excessive blood flow and pulmonary hypertension (C ) profuse perspiration is a sign
(Problem Solving) Musculoskeletal Disorders of heart failure secondary to high level of left ventricular output heart rate and stroke volume are
B -The unique anatomy and circulation of the ends of long bones results in the predilection for localization increased mediated by an increased level of sympathetic nervous system stimulation and activity
of blood borne bacteria. In the metaphysic, nutrient arteries branch into non-anastomosing capillaries thus increasing the circulation of catecholamines combined with increased work of breathing resulting in
under the physics, which make a sharp loop before entering venous sinusoids draining into the marrow. the elevation of in total body oxygen consumption often beyond the oxygen transport ability of the
Blood flow in this area is sluggish and provides an ideal environment for bacterial seeding. circulation (D recurrent respiratory infection secondary to the presence of wet lung syndrome that
(Nelsons Textbook of Pediatrics, 17th ed., 2004, pp. 2297-2298) serves as a niduos infection coupled with the disruption of the mucociliary clearance these will be
responsible for the recurrence of URTI (Nelsons Textbook of Pediatrics, 17 th ed, 2004)
38. It is the most common primary malignant bone tumor in children and adolescents, which
shows a "sunburst" pattern on radiographs: 45. Which of the following cardiac anomaly is NOT present in Tetralogy of Fallot?
A. Ewing sarcoma A. Pulmonary stenosis
B. Osteosarcoma B. ASD
C. Overriding of the aorta (Problem Solving) - Respiratory
D. Right ventricular hypertrophy A - PEFR variability =<20%
(Recall) - Cardiovascular Peak expiratory flow rate variability is a measure of the stability of the airways and it is
B - Atrial septal defect is NOT seen in patients with TOF. It is ventricular septal defect (VSD) considered as a diagnostic tool to predict the success of ones treatment PEFR variability =<20% is still
is the defect that is part of the defect and the VSD is frequently non restrictive and large frequently within normal limits. Intermittent asthma has normal PEFR and PEFR variability values.
located just below the aortic valve. (A) Pulmonary stenosis leads to the obstruction of the Right Symptoms of these patients are very infrequent
ventricular outflow. The pulmonary valve annulus may be of nearly normal size or may be quite small in (B) Once PEFR is 60-79% of the predicted, it only means that airway obstruction is present [(N) PEFR
size. The valve itself is bicuspid and occasionally is the only site of the stenosis. In cases where the = 80% of the predicted] and signals that the patient belongs to the moderate persistent category
right ventricular outflow tract is completely obstructed, pulmonary blood flow may be (C ) FEV1 (forced expiratory volume in one second) is a measure of airflow obstruction. And the value
supplied by a patent ductus arteriosus (PDA) and by major aortopulmonary collateral arteries arising from of <60% indicates that the patient belongs to the category of severe persistent asthma.
the aorta (C ) Over riding of the aorta is part of the congenital defect (D) Right ventricular hypertrophy (D) PEFR variability =20-30% means that airways are still unstable and asthma is not well controlled
is due to the degree of right ventricular outflow obstruction (Nelsons Textbook of Pediatrics, and the patient belongs to the mild persistent asthma category (Nelsons Textbook of Pediatrics, 17 th
17th ed, 2004) ed, 2004)

46. A 3-year old boy was admitted to the ER because of difficulty of breathing. History revealed 51. An 8-year old presents with sneezing, clear rhinorrhea, and nasal itching. P.E. findings show
that he developed high grade fever and sore throat 24 hours prior to consult with associated boggy, pale nasal edema with a clear discharge. The most likely diagnosis is:
difficulty of swallowing. Physical examination showed a very toxic looking boy, highly febrile, A. Foreign body
with labored breathing and hyper extended neck and drooling of the saliva. The most plausible B. Vasomotor rhinitis
diagnosis of the above case is: C. Neutrophilic rhinitis
A. Acute infectious laryngitis D. Allergic rhinitis
B. Acute epiglottitis (Problem Solving) Immunology/Allergology
C. Acute laryngotracheobronchitis D - Allergic rhinitis is often seasonal and associated with allergic conjunctivitis. Eosinophils
D. Acute bacterial tracheitis predominate in the nasal secretions.
(Nelsons Textbook of Pediatrics, 17thed, 2004, pp. 759-760)
(Problem Solving) - Respiratory
B - Acute epiglotittis 52. The mother of 7-year old girl with acute strep throat calls to report that within 15 minutes
This is a potentially lethal condition characteristically presenting with acute fulminating after the first dose of oral penicillin you prescribed, she is complaining of itching and has
course of high grade fever, sore throat, dyspnea and rapidly progressing respiratory obstruction. Drooling developed hives. Which of the following should you recommend?
of the saliva is frequently present and is due to difficulty of swallowing. Hyperextension of the neck is due A. Give her oral antihistamines and call again if not improved within 30
to his attempt to maintain the patency of the airway. This fatal disease is frequently caused by H. minutes
influenzae. B. Bring her to your office or the nearest emergency room
(A) Acute infectious laryngitis is frequently caused by viral agents and the disease is usually mild and non C. Substitute erythromycin for penicillin
fatal. The onset of the disease is usually characterized by an upper respiratory tract infection during D. Bring her to the nearest emergency room once difficulty of breathing
which sore throat, cough and hoarseness appear. Respiratory distress is unusual except in the very is experienced
young infants where the airways are very compliant and small in caliber. (Problem Solving) Immunology/Allergology
(C) Acute laryngotracheobronchitis croup is again frequently caused by viruses. Most of the patients will B - The urticarial reaction described in the question may develop into anaphylaxis which
present with upper respiratory tract infection with a combination of rhinorrhea, pharyngitis, mild cough and requires emergency treatment. Aside from this, penicillin should be stopped and a substitute non-
low grade fever for 1-3 days before the appearance of the signs and symptoms of upper airway penicillin appropriate antibiotic chosen.
obstruction. It starts with barking cough ,hoarseness and inspiratory stridor which characteristically (Nelsons Textbook of Pediatrics, 17th ed, 2004,Chap 137, pp. 781-782)
becoming worse at night and often recurring with decreasing intensity for several days and completely
resolves with in a week 53. A child has abdominal pain, arthritis, microscopic hematuria, and a purpuric rash only on the
(D) Acute bacterial trachietis this entity is a form of bacterial infection of the upper airway and does not lower extremities. Which of the following is the most likely diagnosis?
involve the epiglottis. It is capable of causing life threatening airway obstruction. It is frequently caused by A. Meningococcemia
staphylococcus aureus and other organisms like Moraxella catarrhalis, nontypable H. Influenzae and B. Varicella
anaerobic organisms have been implicated. It frequently occurs in children younger than 3 years of age. C. Henoch-Schonlein vasculitis
A patient seen at the pediatric OPD clinic because of prolonged harsh barky cough that lingered behind D. Post streptococcal glomerulonephritis
after a bout of viral infection not responsive to treatment i.e. bronchodilators and mucolytics and (Problem Solving) Immunology/Allergology
disappears when the patient is asleep. (Nelsons Textbook of Pediatrics, 17th ed, 2004) C -The purpura on the lower extremities suggests Henoch-Schonlein Vasculitis.
Meningococcemia is generalized. Varicella gives papulo-vesicular lesions which are likewise
47. The mechanism of hypoxia in pulmonary edema is: generalized. Erythema nodosum is the cutaneous lesion usually found on the lower extremities in post-
A. V/Q mismatch strep infection (Nelsons Textbook of Pediatrics, 17 th ed, 2004,pp. 794, 826-828)
B. Hypoventilation
C. Diffusion impairment
D. R-L shunt 54. A 2-day old neonate with vomiting of bilous material since birth was brought to your hospital.
(Problem Solving) - Respiratory X-rays taken showed double-bubble.
C - diffusion impairment A. Metabolic problems must be addressed initially
In pulmonary edema there is fluid that acts as a barrier between the alveolo-capillary membrane which B. Schedule for emergency laparotomy
increases the travel time of the O2 from the alveoli to the capillary thus hindering the diffusion of the gas C. Do upper GI series
through the membrane and subsequently lowers theO2 levels in the circulation. D. Intubate once seen
V/Q mismatch as a cause of hypoxemia occurs in two stages: V ventilation when there will be less O2 (Problem Solving) - Neonatology
delivered to the alveoli due to airflow obstruction as it happens in pneumonia Q (perfusion) hypoxemia A - (Nelsons Textbook of Pediatrics,, 17th ed, 2004, pp. 1233-1234)
occurs despite adequate oxygenation if the circulation is blocked as in pulmonary embolism
Hypoventilation could cause low O level due to decreased amount of O2 delivered in to the alveoli due 55. A 24-hour old neonate is brought to the ER because of inability to pass meconium in 24 hours.
to central causes (CNS depression or infection) or due to low levels of O2 in the atmosphere due to He does not have vomiting or distention. Your recommendation would be to:
high altitude A. Do suction rectal biopsy
R-L shunt this happens particularly in patient with cardiac shunts where a high percentage of the cardiac B. Request for barium enema
output returns to the general circulation without passing through the lungs. Or this can occur in cases of C. Request for abdominal x-ray
intrapulmonary shunts as well D. Observe the patient
(Nelsons Textbook of Pediatrics, 17th ed, 2004) (Problem Solving) - Neonatology
D - (Nelsons Textbook of Pediatrics, 17th ed, 2004, pp. 1232-1241)
48. The common infectious cause/s of bronchiectasis is/are:
A. Pertussis 56. A 7-day old 900 gram pre-term has been noted to have abdominal distention with gastric
B. Klebsiella pneumoniae retention. OGT drainage is 10 cc in 24 hours and stool occult blood is positive. Abdominal x-ray
C. Streptotoccus pneumoniae showed pneumatosis intestinalis. This patient should:
D. H. influenzae A. Undergo immediate surgery
(Recall) - Respiratory B. Have intensive medical therapy
A - Pertussis C. Peritoneal drainage
Infections due to Bordatella pertussis, measles, rubella, togavirus, respiratory syncytial D. Have a blood culture.
virus and Mycobacterium tuberculosis induce chronic inflammation, progressive bronchial wall damage (Problem Solving) - Neonatology
and dilatation of the bronchial tree. The common thread in the pathogenesis of bronchiectasis isdifficulty B - (Nelsons Textbook of Pediatrics, 17th ed, 2004, pp. 590-591)
clearing secretions and recurrent infections.
(Nelsons Textbook of Pediatrics, 17th ed, 2004) 57. A live 30 weeks of gestation baby boy was born via cesarean section to a diabetic mother.
Grunting and tachypnea was noted on the 6th hour of life. After receiving therapeutic measures,
49. The pathologic findings of bronchopulmonary dysplasia (BPD) consist of the following: the patient improved. However on the 4th day of life, lethargy, apnea and poor muscle tone was
A. Decreased alveolarization noted. Transfontanel cranial ultrasonography was done and showed increased echogenecity at
B. Decreased alveolar septation the thalamocapsular region with ventricular dilatation. What is the grade of this patients germinal
C. Minimal airway disease matrix hemorrhage?
D. All of the above A. Grade 1
(Recall ) - Respiratory B. Grade 2
D - all of the above C. Grade 3
D. Grade 4
BPD is a result of lung injury in infants requiring mechanical ventilation and supplemental (Problem Solving) - Neonatology
oxygenation. It is apparent that patients with BPD have decreased alveolarization, alveolar septation and D - (Nelsons Textbook of Pediatrics, 17th ed, 2004, pp. 563)
minimal airway disease all of which suggest arrest in lung development. The lung injury occurring in
children is due to an interaction of multiple factors. Since RDS is a disease of progressive alveolar 58. Which of the following chest radiographic findings can be found in mild ventricular septal
collapse, Atelectasis which is affected by insufficient PEEP together with ventilator-induced increased defect?
lung volume and regional overdistention promotes injury. Oxygen promotes injury by producing free A. Small heart
radical that cannot be metabolized by immature antioxidant systems. Therefore, mechanical ventilation B. Increased pulmonary vascularity
and /or oxygen injure the preterm lung by affecting alveolar and vascular development. Moreover, C. Increased size of the aorta
inflammation as measured by circulating neutrophils and macrophage in the alveolar fluid and pro- D. Left atrial enlargement
inflammatory cytokines contribute to the progression of the lung injury. (Problem Solving) - Neonatology
(Nelsons Textbook of Pediatrics, 17th ed, 2004) B - (Nelsons Textbook of Pediatrics, 17th ed, 2004, pp. 1509)

50. A patient is considered to have intermittent asthma when the following is/are present: 59. Meconium aspiration in utero is explained by:
A. PEFR variability = <20% A. Obstruction of fetal airways
B. PEFR 60-79% of predicted B. Chemical property of meconium
C. FEVI <60% C. Intrauterine infection
D. PEFR variability = 20-30% D. Chronic fetal asphyxia
(Problem Solving) - Neonatology 71. Which of the following features is TRUE of all infants born to HIV-infected women?
D - (Nelsons Textbook of Pediatrics, 17th ed, 2004, pp. 547, 583-584) A. They will have low CD4 cell counts
B. They will eventually develop AIDS
60. A key predictor for death or brain damage after an asphyxial episode: C. They will have antibodies to HIV
A. HIE Sarnat Stage 2 D. They will be infected with HIV
B. Failure to establish spontaneous respiration by 15 minutes (Problem Solving) Infectious Diseases
C. Onset of seizure within the first 24 hours of life C - (Nelsons Textbook of Pediatrics, 17th ed., 2004, p. 1161)
D. Establishment of adequate oral feedings by 48 days of life
(Recall) - Neonatology 72. One of the following is LEAST associated with ascaris infection in humans:
B - (Nelsons Textbook of Pediatrics, 17th ed, 2004, pp. 567) A. Intestinal obstruction
B. Loefflers syndrome
61. In a newborn suspected of having choanal atresia, respiratory distress may be relieved by: C. Iron deficiency anemia
A. Opening the mouth D. Pancreatitis
B. Intubation (Recall) Infectious Diseases
C. Bag and mask ventilation C - (Nelsons Textbook of Pediatrics, 17th ed., 2004, pp. 1156)
D. Administering O2
(Problem Solving) - Neonatology 73. Which of the following does NOT RESULT from a vesicoureteral reflux?
A - (Nelsons Textbook of Pediatrics, 17th ed, 2004, pp. 1387) A. Hypertension
B. Chronic failure
62. The most common clinical manifestation of Persistent Pulmonary Hypertension is: C. Proteinuria
A. Respiratory distress D. Dilatation of the ureters
B. Pallor (Problem Solving) Nephro/GU
C. Cyanosis C - Vesicoureteral reflux results from valvular incompetence at the uretero vesicular junction
D. Apnea as a result of a shortened segment of ureter within the bladder wall. It is often associated with other
(Recall) - Neonatology genitourinary anomalies. Vesicoureteral reflux can result in chronic renal failure, dilatation of the ureters,
C - (Nelsons Textbook of Pediatrics, 17th ed, 2004, pp. 585) hypertension and urinary tract infections. Proteinuria results from glomerular injury.
(Nelsons Textbook of Pediatrics, 17th ed, 2004,pp. 1790-1794)
63. Which of the following statements regarding the diagnosis of intrauterine infection is/are
accurate? 74. The triad of microangiopathic hemolytic anemia, renal failure, and thrombocytopenia is
A. IgM in neonatal serum may be used as a screening tool characteristic of which one of the following?
B. Total IgM has a low rate of both false positive and false negative A. Membranous lupus nephritis
results B. Focal glomerulonephritis secondary to sepsis
C. IgM titers may have low specificity and low sensitivity C. Acute post-streptococcal glomerulonephritis
D. IgG rising titers in infancy are not helpful D. Hemolytic-uremic syndrome
(Problem Solving) - Neonatology (Recall) Nephro/GU
A- (Nelson Textbook of Pediatrics, 17th ed, 2004) D -The name gives the answer away.
(Nelsons Textbook of Pediatrics, 17th ed, 2004,pp. 1746-1747)
64. Thyrotoxicosis in the first day of life is most likely to occur in an infant born to a mother
A. With untreated hypothyroidism 75. A 2-year old male developed an upper respiratory tract infection that was followed in 2 weeks
B. With untreated Graves disease by general edema. His blood pressure is normal. Urinalysis reveals 2-5/hpf and +4 protein. His
C. With Graves disease being treated with antithyroid medications BUN is 19 mg/dl, creatinine 0.6 mg/dl, cholesterol 402 mg/dl, serum albumin 0.9 g/dl, ASO=200,
D. Receiving iodides as therapy for chronic Thyrotoxicosis and C3=92 mg/dl. The most likely diagnosis would be:
(Problem Solving) - Neonatology A. Poststreptococcal glomerulonephritis
B - (Nelsons Textbook of Pediatrics, 17th ed, 2004, pp. 1886) B. Membranous glomerulonephritis
C. Minimal lesion nephrotic syndrome
65. A 6-year old male was brought to the OPD because of jaundice of 5 days associated with D. Focal sclerosis
anorexia of one week. A hepatitis profile done on him revealed the following: (Problem Solving) Nephro/GU
Anti-HAV (IgM) - Reactive C - Hypoalbuminemia, proteinuria, edema and hyperlipidemia constitute the nephrotic
HBsAg - Non-reactive syndrome. Hypertension, azotemia, edema or hematuria would suggest nephritis but may also be
Anti-HBc - Non-reactive encountered in minimal lesion nephrotic syndrome. This patient has nephrotic syndrome, not nephritis.
Anti-HBe - Non-reactive (Nelsons Textbook of Pediatrics, 17thed, 2004,pp. 1740-1746 ; 1753-1757)
Anti-HBs - Non-reactive
The patient Had a recent: 76. The recommended age to perform corrective surgery in a child with a unilateral undescended
A. HAV and HBV infection testes is:
B. HAV infection with a post infection of HBV A. The first 6 months of life
C. HAV infection with HBV immunity B. Between 21 and 18 months of age
D. HAV infection and post infection of HAV C. Between 5 and 7 years of age
(Problem Solving) Infectious Diseases D. Before puberty
C (Nelsons Textbook of Pediatrics, 17th ed., 2004, pp. 1324-1329) (Recall) Nephro/GU
B - Corrective surgery performed between 12 and 18 months of age represents a safe
66. A 9-month old female was brought to the OPD because of watery diarrhea, yellowish, non- balance between anesthetic risk, allowance of time for the testes to descend, and the risks of leaving a
bloody, non-mucoid stools, with no pus nor RBCs. Her anterior fontanel is slightly sunken. The testis in the abdomen. (Nelsons Textbook of Pediatrics, 17thed, 2004,pp. 1817-1820)
most likely organism to cause this type of diarrhea is:
A. ETEC 77. Maternal varicella results in severe neonatal varicella when maternal infection takes place
B. Vibrio cholera during:
C. Rotavirus A. The 1st trimester
D. Norwalk virus B. The 2nd trimester
(Problem Solving) Infectious Diseases C. The 3rd trimester
C (Nelsons Textbook of Pediatrics, 17th ed., 2004, pp. 1324-1329) D. The week before and after delivery
(Problem Solving) Infectious Diseases
67. Which of the following is TRUE of candidal infection? D - Birth within 1 week before or after the onset of maternal varicella frequently results in
A. Diaper dermatitis is the most common infection caused by candida the newborn developing varicella, which may be severe. The risk to the newborn is dependent on the
B. With improved survival of very LBW infants, candidemia has become less amount of maternal anti-VZV antibody that the fetus acquired transplacentally before birth. If the internal
frequent n NICUs between maternal chickenpox and parturition is less than 1 week, the newborn will be unlikely to have
C. Fluconazole is the drug of choice for the treatment of systemic candiciasis protective VZV antibody and neonatal chickenpox may be exceptionally severe.
D. Most cases of candidemia is Immunocompromised patients are due to non- (Nelsons Textbook of Pediatrics, 17th ed., 2004, p. 1058)
candida
albicans spp. 78. A 2-year old has a positive tuberculin test (15 mm induration). Which of the following would
(Problem Solving) Infectious Diseases be suggestive of military tuberculosis?
A - (Nelsons Textbook of Pediatrics, 17th ed., 2004, pp. 1012-1013) A. Fever
B. Hepatosplenomegaly
68. Which of the following statements is NOT TRUE of Amebiasis? C. Hilar adenopathy on chest x-ray
A. The infective stage is the Entamoeba histolytica cyst D. Cough
B. The pathogenic stage is the Entamoeba histolytica trophozoite form (Problem Solving) Infectious Diseases
C. A carrier of E. histolytica cysts should be treated B - Miliary tuberculosis suggests lymphohematogenous spread or disseminated form of
D. It is the most common cause of bloody stools TB, occurring is distant sites, including liver, spleen, skin and other organs aside from the lungs.
(Recall) Infectious Diseases Fever and cough are non-specific manifestations of TB which may be found in other diseases, while hilar
D - (Ref. Nelsons Textbook of Pediatrics, 17th ed., 2004, pp. 1123-1125) adenopathy on chest x-ray, may be found in primary pulmonary TB.
(Nelsons Textbook of Pediatrics, pp. 962-964)
69. Which of the following diseases has the greatest capacity to be a pandemic?
A. HIV 79. A 3-year old nonimmunized child is seen at the OPD and diagnosed as having measles. There
B. Influenza is an 8-month old nonimmunized sibling at home. Appropriate management of this sibling would
C. Measles include:
D. Hepatitis B A. A modifying dose of gammaglobulin
B. A preventive dose of gammaglobulin
(Recall) Infectious Diseases C. Immediate immunization with live attenuated measles vaccine
B - (Nelsons Textbook of Pediatrics, 17th ed., 2004, pp. 1072-1074) D. Immediate immunization with killed measles vaccine
(Problem Solving) Infectious Diseases
70. A newborn was found to have the following anomalies: hydrocephalus, cicatricial scarring A - Passive immunization with immune globulin is effective for prevention and attenuation
over the 6th-7th left intercostals, malformed feet (fusion and maldevelopment of both feet). Which of measles within 6 days of exposure. Susceptible household and hospital contacts who are <12 months
congenital infection shows these findings at birth? of age or who are pregnant should receive immune globulin (modifying dose, 0.25 ml/kg; maximum 15
A. HIV ml) IM as soon as possible after exposure, but within 5 days. Immunocompromised persons should
B. CMV receive immune globulin (preventive dose, 0.5 ml/kg ; maximum 15 ml) IM regardless of immunization
C. Parovovirus B19 status.
D. Varicella-Zoster virus
(Recall) Infectious Diseases 80. An 8-year old male consulted the OPD because of high grade fever and sore throat. The
D - (Nelsons Textbook of Pediatrics, 17th ed., 2004, p. 1059) pertinent P.E. findings were: hyperemic oropharynx, enlarged tonsils with exudates, petecchiae
on the soft palate and painful, enlarged cervical lymphadenopathies. This patient is most likely 86. The findings of sudden onset of fever, petecchial rash, and BP = 70/50 in a 5 year old child is
suffering from: most suggestive of infection with:
A. Streptococcus pyogenes A. Neisseria meningitidis
B. Epstein Barr Virus B. Hemophilus influenzae
C. Adenovirus C. Staphylococcus aureus
D. Corynebacterium diphtheria D. Streptococcus pneumoniae
(Recall) Infectious Diseases (Problem Solving) - Infectious Diseasess
C - (Nelsons Textbook of Pediatrics, 17th ed., 2004, pp. 870-879) A - These findings are compatible with meningococcemia caused by Neisseria meningitides
which can progress rapidly over hours to septic shock. The other etiologic agents, while they may
81. An 8-month old boy presents with failure to thrive, thrush, lymphadenopathy, and cause a similar picture these are usually in relation to DIC which would manifest over a longer period of
pneumocystis carinii pneumonia. He most likely has: time, not in a matter of hours.
A. Severe malnutrition (Nelsons Textbook of Pediatrics, 17th ed, 2004, p. 897)
B. Acute leukemia
C. HIV infection 87. Of the following parasitic infections, which is most likely to present with intestinal
D. X-linked hypogammaglobulinemia obstruction?
(Recall) - Infectious Diseases A. Trichuris trichura
C - These constellation of manifestation are associated with HIV infection. The clinical B. Necator americanus
manifestations of HIV infection vary widely among infants, children, and adolescents. In most infants, PE C. Ascaris lumbricoides
findings at birth are normal. Initial symptoms are subtle, such as lymphadenopathy and D. Enterobius vermicularis
hepatosplenomegaly, or non-specific such as failure to thrive, chronic or recurrent diarrhea, interstitial (Recall) - Infectious Diseases
pneumonia, or oral thrush, and may be distinguishable only by their persistence. Whereas systemic and C - A large mass of Ascaris lumbricoides leads to intestinal obstruction. Rectal prolapse is
pulmonary findings are common in the United States and Europe, chronic diarrhea, wasting, and severe associated with Trichuris trichura. Heavily infected children with Necator americanus suffer from
malnutrition predominate in Africa. Symptoms found more commonly in children than adults with HIV intestinal blood loss resulting in iron deficiency, which can lead to anemia as well as protein malnutrition.
infection include recurrent bacterial infections, chronic parotid swelling, lymphocytic interstitial Pruritus ani is associated with enterobiasis.
pneumonitis (LIP), and early onset of progressive neurologic deterioration. The HIV classification system (Nelsons Textbook of Pediatrics, 17th ed., 2004, pp. 1156-1159)
is used to categorize the stage of pediatric disease by using 2 parameters: clinical status, and degree of
immunologic impairment (absolute CD4 lymphocyte count or the percentage of CD4 cells). 88. A 10-month old child has a temperature of 39-40C for 4 days without other signs. On the 4 th
Category A (Mild Symptoms)Children with at least 2 mild symptoms such as: day, a maculopapular rash appears, and the temperature returns to normal. The most likely
- lymphadenopathy diagnosis is:
- parotitis A. scrub typhus
- hepatomegaly B. roseola
- splenomegaly C. rubeola
- dermatitis D. echoviral infection
- recurrent or persistent sinusitis or otitis media (Recall ) - Infectious Diseases
B - (Nelsons Textbook of Pediatrics, 17th ed., 2004, pp. 1069-1072)
Category B (Moderate Symptoms):
- lymphocytic interstitial pneumonitis (LIP) 89. Which of the following is the chemoprophylactic antimicrobial given to intimate contacts of a
- oropharyngeal thrush persisting for <2 months 7-year old with meningococcal meningitis?
- recurrent or chronic diarrhea A. Rifampin
- persistent fever >1 month B. aqueous Pen G
- hepatitis C. Isoniazid
- recurrent herpes simplex stomatitis or HSV esophagitis or D. Erythromycin
pneumonitis (Problem Solving) - Infectious Diseases
- disseminated varicella (i.e., with visceral involvement) A - (Nelsons Textbook of Pediatrics, 17th ed., 2004, pp. 898-899)
- cardiomegaly
- nephropathy 90. The clinical manifestations in DHF are secondary to the pathogenetic mechanism of:
Category C (Severe Symptoms): A. direct invasion of the virus to the different organ system
Children with 2 serious infections (i.e., sepsis, meningitis, B. hypersensitivity reaction
pneumonia) in a 2 year period C. immune enhancement
- esophageal or lower respiratory tract candidiasis D. adherence of the viruses to the endothelial cells
- cryptococcosis (Recall) - Infectious Diseases
- cryptosporidiosis (>1 mo) C - (Textbook of Pediatrics & Child Health, Del Mundo, et al (eds.), 4th ed, 2000, p. 561)
- encephalopathy
- malignancies 91. In a case of DHF (Grade 3) who is bleeding profusely, which of the following fluids (all
- disseminated myocobacterial infection available) would you give?
- pneumocystis carinii pneumonia (PCP) A. properly typed & X-matched fresh whole blood
- cerebral toxoplasmosis (onset after 1 month of age) B. properly typed & X-matched fresh plasma
- severe weight loss C. D5LRS
The Pneumocystis carinii pneumonia (PCP) is the most common opportunistic D. D5NSS
infection in the pediatric population. The peak incidence of PCP occurs at age 3-6 months with the (Problem Solving) - Infectious Diseases
highestmortality rate in children <1 year of age. A - (Textbook of Pediatrics & Child Health, Del Mundo, et al (eds.), 4 th ed, 2000, p. 569 -
(Nelsons Textbook of Pediatrics, 17th ed., 2004, pp. 1112-1113) 571)

82. In our country, the first dose of live attenuated measles vaccine should be administered:
A. at 4 months of age 92. The following CSF analysis results are compatible with which of the following clinical
B. at 6-9 months of age entities?
C. at 12-15 months of age CSF Results: Opening pressure = 300 mm H2O
D at 18-24 months of age WBC = 296 (segs: 10% ; lymphos : 90%)
(Recall) - Infectious Diseases Protein = 2 g/L
B - The attack rate of measles among the susceptibles in <1 year is 80% (DOH Philippines CSF sugar / RBS = 20%
2000), such that even if vaccine efficacy at 6 months is just 50% and at 9 months it is 85%, we give it as A. Acute meningococcal meningitis
early as 6 months, during which time transplacentally acquired maternal antibodies for measles would B. TB meningitis
already be at its nadir (DOH Philippines 2000) C. Japanese B encephalitis
D. Febrile seizures
83. Neonatal bacterial sepsis is most commonly caused by which one of the following organisms (Problem Solving) Infectious Diseases
(in the Philippines): B - (Nelsons Textbook of Pediatrics, 17th ed., 2004, pp. 965, 2040-2044)
A. Group B streptococci
B. Pseudomonas aerugenosa 93. On the 3rd day of treatment for Hemophilus influenzae meningitis, an eight month old child
C. Streptococcus pneumoniae who had been alert is noted to be lethargic. Serum electrolytes reveal the following:
D. E. coli Na = 120 mEq/L
(Recall) - Infectious Diseases Cl = 83 mEq/L
D - In our country, gram (-) organisms are the most common causes of neonatal bacterial K = 3.1 mEq/L
sepsis (2:1) compared with the gram positive organisms. The gram negative enteric bacilli like E. coli BUN = 2 mg/dl
are the more common causative agents. Pseudomonas aerugenosa, a gram negative bacilli, is a The most likely cause of the lethargy and hyponatremia in this patient:
common nosocomial pathogen. A. Acute renal failure
(Textbook of Pediatrics and Child Health, Del Mundo, Fe et. al (eds.) 4 th ed, 2000, p. B. Congestive heart failure
265) C. Syndrome of inappropriate ADH secretion
D. Subdural effusions
84. The first clinical manifestation of tetanus neonatorum usually is: (Problem Solving) Infectious Diseases
A. Fever C - (Nelsons Textbook of Pediatrics, 17th Ed, 2004, p. 200)
B. Vomiting
C. Spasms 94. A 28 year old primigravida mother, who had a past history of adequately treated PTB, gave
D. Difficulty sucking and swallowing birth to a healthy 3 kg baby. Which of the following preventive measures against TB would you
(Recall) - Infectious Diseases take?
D - Neonatal tetanus is generalized in type and starts as progressive difficulty in sucking A. Separate the baby from the mother for another month
and irritability. (Textbook of Pediatrics and Child Health, Del Mundo, Fe, et al (eds.), 4th ed, 2000, p. 469) B. Separate the baby from the mother and give INH for 3 months
C. Give BCG only
85. The usual course of pertussis in an infant is characterized by: D. Give BCG now but separate the baby from the mother for a month
A. 4 5 days of high grade fever followed by cough and whooping (Problem Solving) Infectious Diseases
B. sudden onset of fever, cough, and whooping C - (Textbook of Pediatrics & Child Health, Del Mundo, et al (eds.), 2000 pp. 516-525)
C. rhinitis and possibly low grade fever, followed by gradual worsening of cough and
finally whooping 95. A 2-year old boy was admitted because of low to moderate grade fever of 3 weeks, on and off
D. gradual onset of cough, followed by abrupt onset of fever and whooping frontal headache of 1 week, squinting of 1 day, one episode of generalized seizure of 2 minute
(Recall) - Infectious Diseases duration 6 hours prior to admission. No medical consult done. No medications given except
C - Pertussis is a 6-week disease, divided into catarrhal (congestion, rhinorrhea, then low paracetamol. Which of the following clinical consideration is NOT COMPATIBLE with this history?
grade fever, lacrimation, conjunctival suffusion), paroxysmal, and convalescent stages. A. Acute meningococcal meningitis
(Nelsons Textbook of Pediatrics, 17th ed., 2004, p. 909) B. TB meningitis
C. Cryptococcal meningitis
D. Brain abscess of otogenic origin After 4-hours, he was noted to have severe vomiting and diarrhea. A number of people
(Problem Solving) Infectious Diseases who attended the party suffered the same fats. The most probable diagnosis is:
A - (Nelsons Textbook of Pediatrics, 17th ed., 2004, pp. 965, 2040-2044) A. Streptococcus B. Staphylococcus C. Mycoplasma D.
Pneumococcus
For Nos. 96 to 100: -2-
Mario is 2 years old, brought to the Well Child Clinic for his regular visit. On A 11. A 5-year-old child was noted to have spiking fever associated with conjunctivitis. He also
physical exam, he had the following anthropometric measurements: Weight - 15 kgs, Height 90 complained of calf pain and develop jaundice in five days time after onset of symptoms. I
cms, Head Circumference 48 cms. On developmental screening, he was able to do the would be highly considering:
following: could stand momentarily on one foot, draw a circle and imitate a cross, talk in simple A. Leptospirosis B. Hepatitis C. Measles D.
sentences and tell a story. Mother claimed that he is dry by night. Chlamydial infection
A 12. Bleeding into the adrenals with septicemia of meningococcemia is seen in:
96. Describe the nutritional status of Mario: A. Waterhouse-Friderichsen Syndrome C. Job-Chedeiki
A. not stunted, not wasted Sydnrome
B. mildly stunted, not wasted B. Wiskott-Aldrich Syndrome D. Nephrotic Syndrome
C. not stunted, mildly wasted B 13. A child had an accident sustaining wounds from a rusty iron bar. After 14 days, he
D. Mildly stunted, mildly wasted developed trismus, generalized stiffness and spasms. He was however oriented to time,
(Problem Solving) - Neurodev place and person. The most probable disease condition affecting the patient is:
A - (Textbook of Pediatrics & Child Health, Del Mundo, et al (eds.), 4 th ed, 2000, pp. 78-80) A. Rabies C. seizure disorder
secondary to CNS infection
97. The head circumference of Mario is: B. Tetanus D. DPT shot
A. normal
B. below normal F. RESPIRATORY
C. above normal
D. variation of normal A 14. Refers to the volume of normal breathing:
(Problem Solving) Neurodev A. Tidal volume C. Inspiratory reserve volume D. Expiratory reserve
A - (Textbook of Pediatrics & Child Health, Del Mundo, et al (eds.), 4 th ed, 2000, p. 80) volume
98. Based on developmental screening, his development is: B. Vital capacity E. Residual volume
A. normal C 15. A condition characterized by progressive granulation of the small airways:
B. delayed A. bronchial asthma C. bronchiolitis obliterans D.
C. advanced bronchiectasis
D. deviant B. acute bronchitis E. Acute bronchiolitis
(Problem Solving) - Neurodev A 16. The following is/are the characteristics of hydrocarbon aspiration pneumonia:
C - (Textbook of Pediatrics & Child Health, Del Mundo, et al (eds.), 4th ed, 2000, pp. 83-112) A. hydrocarbon has a low viscosity and a high volatility properties
B. asymptomatic patient can be send home immediately from the ER
99. The approximate developmental age of Mario is: C. routine antibiotics should be given to prevent secondary infection
A. 2 years old D. All
B. 2 years old C 17. A condition resulting from transudation of fluid from the pulmonary capillary into the
C. 3 years old alveolar spaces and bronchioles:
D. 3 years old A. Atelectasis C. Pulmonary edema D.
(Problem Solving) Neurodev Pulmonary infarction
C - (Textbook of Pediatrics & Child Health, Del Mundo, et al (eds.), 4th ed, 2000, pp. 83-112) B. Pulmonary Effusion E. Emphysema
C 18. A condition characterized by congenital flabbiness of the epiglottis and subglottic
100. At 2 years of age, Mario had a height of 85 cms. How tall would he be when he becomes an aperture:
adult? A. Tracheoesophageal fistula
A. 4 feet 6 inches B. Laryngeal webs
B. 5 feet C. Laryngomalacia
C. 5 feet 6 inches D. GER
D. 6 feet E. Laryngeal Atresia
(Recall) - Neurodev C 19. The following condition(s) should be considered in a child with unilateral foul smelling nasal
C - (Textbook of Pediatrics & Child Health, Del Mundo, et al (eds.), 4th ed, 2000, pp 78-80) discharge:
A. Acute rhinitis
B. Allergic rhinitis
C. Nasal foreign body obstruction
DEPARTMENT OF PEDIATRICS Fatima medschool D. Nasal polyps
E. Sinusitis
CARDIO-ENDO A 20. The most common cause of obstructive sleep apnea in children (OSA)
A. Tonsillar hypertrophy
D 1. In diabetes insipidus, the patients urine is of: B. Obesity
A. high specific gravity, less volume C. low specific gravity, less volume C. Hypotonia
B. high specific gravity, large volume D. low specific gravity, large volume D. Acute Tonsillitis
D 2. In endocrine disease: E. Retropharyngeal abscess
A. an end-organ defect responds to exagenous hormone
B. all lesions present at birth G. VIRAL INFECTION
C. all features of disease are not reversible
D. hormone replacement is one of its mode of management B 21. Clinical features of a classic congenital rubella syndrome, EXCEPT:
D 3. The diastolic duration of the cardiac cycle is longer than the systolic time because: A. mental retardation
A. contraction of the heart is time-related B. Hydrocephalus
B. the parasympathetic system of the heart is dominant C. Cataracts
C. the heart needs short and vigorous contraction to increase the stroke volume D. Congenital Heart Anomalies
D. the heart needs to relax to increase the LV end-diastolic volume C 22. Lymphadenopathy, may be generalized most commonly suboccipital, post auricular and
D 4. Crying, the first independent respiration of the Neonate produces: post
A. Immediate closure of all intra and extracardiac shunts cervical nodes:
B. Reversal of left-to-right shunts A. Roseola Infantum
C. Systemic vasoconstriction B. Rubeola
D. Increased arterial oxygen saturation C. Rubella
B 5. The pressure change responsible for the foramen ovale flap closure is due to: D. Varicella
A. increased RA volume C. B 23. In passive immunization for rabies, there is a high incidence of serum sickness:
increased RA pressure A. Human rabies serum globulin
B. increased pulmonary venous return D. increased RV B. Animal rabies serum
pressure C. Duck Embryo Vaccine
C 6. The cyanosis in Tetralogy of Fallot: D. Human Diploid Cell Vaccine
A. always evident at birth A 24. Suggestive of Dengue Infection, EXCEPT:
B. is due to intrapulmonary right-to-left shunting A. Hypertension
C. is due to the overriding of the aorta and pulmonic stenosis B. Hyperpyrexia
D. is primarily due to the VSD C. Hepatomegaly
C 7. True of Rheumatic Fever D. Hemorrhagic Manifestation
A. Preceeded by any streptococcal infection C 25. Recommended fluid therapy for DSS:
B. Usually present with high grade fever A. platelet concentrate
C. May present with migratory arthritis involving large joint leaving no sequelae B. D50.3 NaCl
D. May present with migratory polyarthritis leaving a joint deformity C. Plasma expander
D. FWB
E. BACTERIAL INFECTION
H. HEMATOLOGY
A 1-week old male referred to you. Patient was born preterm, developed RDS
B 8. One of the following statements about the treatment of pertussis is true: (Respiratory Distress Syndrome) hence was intubated. At present patient has prolonged profuse
A. Exposure to the sea is conventional therapy bleeding on all punctures sites:
B. Ampicillin do not shorten the duration but decreases the communicability
C. The organism is eradicated in the first twenty four hours after intake of antibiotic B 26. What is the most likely diagnosis:
D. The current recommended therapy is 7 days A. Hemorrhagic Disease of the Newborn
C 9. This organism is motile gram positive anaerobic rods that forms terminal spores B. Disseminated Intravascular Coagulation
resembling drumsticks. The most common disease presentation is that of: C. Idiopathic Thrombocytopenic Purpura
A. Pharyngitis D. Hemophilia
B. Tetanus E. Hemolytic Anemia
C. Pseudomembrane formation B 27. What is/are the most likely pathophysiologic event behind the patients condition?
D. Pleural Effusion A. there is no carboxylation of specific clotting factors
B 10. A 9-year-old kid attended a kiddie party. He was noted to eat salad, noodles and chicken. B. there is tissue necrosis due to widespread fibrin deposition
C. there is transient deficiency of Vitamin K dependent factors
D. there is formation of antibodies of specific clotting factors should contain:
D 28. If needed, what is/are the blood component which the patient urgently requires: A. 100 meq/L of sodium
A. Packed red blood cell B. 75 meq/L of sodium
B. Fresh frozen plasma C. 50 meq/l of sodium
C. Platelet concentrate D. 25 meq/L of sodium
D. Fresh whole blood
E. Granulocyte concentrate C 45. In Hypertonic dehydration 40-50 meq/L of potassium is provided in the IV fluids not only
to replace potassium losses but also:
A 10-year-old MALE admitted due to pallor of 1-month duration. History reveals that his A. to hasten entry of water into the cells
father and his older brother had an unrecalled/abdominal operation when they were on B. to prevent Ileus
school age (8-10 years old) PPE: ABW - 20 kg.; (+) splenomegaly. C. to correct existing metabolic acidosis
D. to prevent hypoglycemia
A 29. The following condition(s) should be included in your clinical impression(s) of the case: C 46. The following conditions will give rise to a state of Hyponatremia, EXCEPT:
A. Hereditary Spherocytosis A. SIADH
B. Thallasemia B. Acute Renal Failure
C. ITP C. Congenital Adrenal Hyperplasia
D. Aplastic Anemia D. Cystic Fibrosis of the Pancreas
E. Congenital Vitamin K deficiency B 47. Under normal condition, the daily water expenditure is about 50% is lost in the urine
A 30. The following is/are true of Thallasemia: 35% is lost in the skin sweat and:
A. There exist heterogenous clinical conditions of the illness A. 15% is lost in the lungs
B. Opportunistic infections are common complications B. 10% is lost in the lungs
C. Transmission of illness is autosomal dominant C. 5% is lost in the lungs
D. It may result from idiosyncratic reaction to drugs D. None of the above
E. All of the above C 48. A 10 kg. patients came in shock with severe Hypotonic Dehydration. The initial hydrating
fluid you should give is:
I. NEUROLOGY A. D50.45 SS 500 cc
B. Normosol-M 500 cc
D 31. A 4-year-old child with Tuberculous Meningitis 2nd stage will most likely develop one of the C. D5LR 500cc
following complications: D. Plasma 500 cc
A. deafness A 49. The following IV fluids do not contain potassium, EXCEPT; which of the following IV
B. ventriculitis fluids contain potassium:
C. blindness A. D5 Ionosol-MB
D. hydrocephalus B. Ringers Lactate
A 32. A 2-week-old infant with bacterial meningitis usually would exhibit: C. Normal Saline Solution
A. lethargy D. D50.45% NaCl Solution
B. opisthotonus D 50. Infants has a higher metabolic rate, about twice the adult and his daily water turnover is
C. headache about:
D. nuchal rigidity A. 15% of his total body water
B 33. Partial seizures may have one of the following features: B. 20% of his total body water
A. are seen only during the infancy period C. 20% of his body weight
B. may or may NOT be associated with loss of consciousness D. 20% of his body surface are
C. are short duration most of the time
D. not seen in the neonatal period L. NEWBORN
C 34. A 4-year-old male child developed easy fatigability becoming more prominent in the late
afternoon. Physical examination shows bilateral ptosis. What is the most likely diagnosis? A 51. Meconium is expelled:
A. Spinal Muscular Atrophy A. within 24-hours after birth
B. Beckers Dystrophy B. 2 days after birth
C. Myasthenia Gravis C. one week after birth
D. Polymyositis D. one hour after birth
B 35. A 6-year-old female child develops diabetes insipidus with severe headache and vomiting. B 52. A 6-10% weight loss is observed in the newborn infant on:
She most likely has a: A. the first day of life
A. Supratentorial tumor B. one week after birth
B. Suprasellar tumor C. 2nd week of life
C. Infratentorial tumor D. 3rd week of life
D. Midline tumor A 53. The most important maneuver to assess muscle tone is:
B 36. A 10-year-old child with Duchennes Muscular Dystrophy is most likely: A. traction maneuver C. horizontal suspension
A. able to walk with a stable gait B. parachute maneuver D. vertical suspension
B. Wheelchair bound
C. Bedridden B 54. Respiratory Distress Syndrome is most likely to occur in one of the following:
D. Able to run A. adequate for gestational age babies
B. Preterm babies
A 37. Joshua is a 9-year old male child who developed moaning sound with tonic clonic C. Postmature babies
contractions of extremities while has was asleep. What is your primary consideration? D. Babies of diabetic mothers
A. Rolandic Epilepsy C 55. One of the following is true of Caput Succedaneum:
B. Complex Partial Seizures A. Does not cross suture lines
C. Absence B. Associated with fracture
D. Akinetic Seizures C. Transient event which disappears in a few days of weeks
D. Associated with increased intracranial pressure
A 56. One of the following events happen after birth:
J. NEPHROLOGY A. Placental circulation ceases when cord is clamped
B. Respiration occurs
A 38. Among children, it is the most common histologic finding in patients with Nephrotic C. Infants expand lungs
Syndrome: D. Foramen ovale closes
A. minimal change nephrotic syndrome C 57. A 36-weeks old infant assumes one of the following:
B. membranous nephropathy A. Hypertonic posture
C. membranoproliferative nephrotic syndrome B. Popliteal angle of 90 degrees
D. focal segmental glomerulosclerosis C. Has elbows slightly passing the midline
B 39. One of the following statements about hypertension is true: D. Fisted hands
A. The most common cause of hypertension in children is essential A 58. It is universally accepted that Hyaline Membrane Disease of the newborn:
B. The most common endocrinologic cause of hypertension is pheocromocytoma A. Is also known as Respiratory Distress Syndrome
C. The appropriate cuff for age covers one third of the upper arm B. Is a leading cause of death among term newborn
D. The arm while taking hypertension should be at the level of the scapula C. Seen only in premature babies
D 40. The most common renal consult in the world is: D. Does not occur in infants of diabetic mothers
A. Acute glomerulonephritis A 59. A preterm is a live newborn delivered:
B. Henoch Schoenlein Purpura A. before 37 weeks gestation
C. IgA Nephropathy B. after 37 weeks gestation
D. UTI C. before 16 weeks gestation
B 41. The following statement is true of Idiopathic Hypercalciuria: D. after 41 weeks gestation
A. It may present as recurrent proteinuria
B. Oral thiazides can normalize urinary calcium excretion PREVENTIVE PEDIATRICS
C. Patients have elevated serum calcium
D. It is familial C 60. A solution of antibodies derived from the serum of animals immunized with specific antigen
B 42. Persistently low C3 with hard to control hypertension is seen among patients with: is:
A. Acute glomerulonephritis A. vaccine
B. Membranoproliferative disease B. toxoid
C. Systemic Lupus Erythematosus C. antitoxin
D. Bacterial Endocarditis D. immuneglobulin
B 43. In Acute Renal Failure, one of the following is usually observed: A 61. Which of the following is a contraindication to vaccine?
A. Small for age kidneys A. Moderate illness with fever
B. Appropriate Bone Aging B. Breastfeeding
C. Enlarged Heart C. Diarrhea
D. Stunted Growth D. Prematurity
B 62. Aluminum hydroxide enhances immunogenicity. What is it?
A. suspending fluid
K. FLUIDS/ELECTROLYTES B. adjuvant
C. preservative
B 44. In mild Hypotonic Dehydration, the parenteral hydrating fluid of choice is the one that D. stabilizer
D 63. If the buttocks are to be used for vaccination, use only the: C 80. Resolution of the oedipal complex should take place by the:
A. central region C. upper inner quadrant A. Anal stage
B. lower outer quadrant D. upper outer quadrant B. Phallic Stage
C. Latency Stage
A 64. Arthritis after MMR vaccination is due to which component? D. Adolescent Stage
A. Rubella A 81. Aspect of behavior most closely related to intelligence:
B. Measles A. Adaptive
C. Mumps B. Language
D. Measles and Rubella C. Fine motor
A 65. One of the following is a usual occurrence after a BCG response: D. Personal and social
A. appearance of wheal B 82. Abdominal circumference is measured at the level of;
B. induration after 2-3 days A. xiphoid bone
C. pustule after 5 7 days B. the plane of the umbilicus
D. scar after 2 3 weeks C. 2 cm below the xiphoid level
B 66. If not administered simultaneously, a 4 week minimum interval is needed for these D. 2 cm below the umbilicus
vaccines: C 83. A seven year old height:
A. DPT and Hepatitis B A. 105 cm
B. MMR and Varicella B. 110 cm
C. Hepatitis B and MMR C. 115 cm
D. OPV and MMR D. 120 cm

M. B 84. True of sensorimotor stage of Piagets Theory, EXCEPT:


N. PEDIATRICS DIAGNOSIS/GENETICS A. object permanence
B. egocentricism
C. causality
A 67. Part of the body that should be examined first when the child is resting and cooperative: D. spatial relationship
A. Lungs C 85. A five year old is able to do the following, EXCEPT:
B. Eyes A. copies square
C. Oropharynx B. count to ten
D. Heart C. adds and subtracts
D 68. The cardiac rate of a child is better counted with: D. uses knife
A. palpation of the wrist C 86. The peak height velocity for boys occur at Tanners:
B. palpation of the apex A. SMR 2
C. visual chestwall inspection B. SMR 3
D. Auscultation C. SMR 4
A 69. The chromosomal abnormality of Edwards Syndrome is due to: D. SMR 5
A. Addition of one chromosome
B. Deletion of Chromosome Q. GIT
C. Breakage of Chromosome
D. Abnormality of sex chromosome D 87. An important factor in many cases of Necrotizing Enterocolitis is:
A 70. In taking a patient history of present illness, informations should begin with: A. Maternal malnutrition
A. nature and date of onset B. Genetics
B. medications patient taken C. Environment
C. Immunization patients received D. Neonatal Anoxia follows delivery
D. Family living condition D 88. The so called common shaped ileum demonstrated on X-ray is a significant finding of:
C 71. Composition of milk formula, amount and interval with childs food intake informations A. ulcerative colitis
should be listed in: B. necrotizing enterocolitis
A. maternal history C. Hirschsprungs Disease
B. birth history D. None of the above
C. feeding history A 89. A condition characterized as having a tender sausages-shaped mass felt in the region of
D. Growth and Development History ascending or transverse colon is seen in:
C 72. This diagnosis should be considered in a child with prolonged unilateral, foul smelling nasal A. Intussusception
discharge: B. Volvulus
A. Allergic Rhinitis C. Meckels Diverticulum
B. Acute Nasopharyngitis D. None of the above
C. Foreign body nasal obstruction B 90. The cardinal signs and symptoms of intestinal obstruction include all best one:
D. Nasal Polyps A. Failure to pass flatus C. Vomiting
B. Failure to pass meconium D. Regurgitation
O. NUTRITION
B 91. The most common virus causing diarrhea in children is:
D 73. True of minerals, EXCEPT: A. Reovirus B. Rotavirus C. Adenovirus D. Enterovirus
A. Sodium is the major regulator of plasma osmolality A 92. All but one are characteristic features of Hepatitis B:
B. Potassium is the principal intracellular cation A. It is a single stranded RNA virus
C. Magnesium is the principal cation of soft tissue B. All age groups affected
D. Calcium retained in the body 50% C. Fever less common
E. Iron intake 90% excreted in the stools D. Can infect the fetus or newborn
D 74. Which vitamin is incorrectly matched with its advisable daily intake for the infant? C 93. The presence of a hard oblong mass along the right rectus muscle is a typical physical
A. Vitamin A 1800 IU finding seen in:
B. Vitamin D 400 IU A. Intussusception
C. Vitamin C 75 mg B. Peptic Ulcer
D. Niacin 0.6 mg C. Congenital Hypertrophic Pyloric Stenosis
E. Thiamine 0.5 mg D. None of the above
B 75. Tryptophan load test is a screening for deficiency of: A 94. True of Tracheoesophageal Atresia/Fistula type B, EXCEPT:
A. Thiamine A. Abdominal distention
B. Pyridoxine B. Drooling of saliva
C. Folacin C. Chocking and coughing
D. Cobalamine D. (+) history of maternal polyhydramnios
E. Niacin D 95. The most common type of Hypoplasia of the mandible is:
B 76. Protein content of this type of milk is much higher than breast milk about 21% vs 7-16%: A. Vincents Angina C. Trench Mouth
A. evaporated milk B. Pyorrhea D. Pierre-Robin Syndrome
B. whole cows milk
C. condensed milk R. ALLERGY/IMMUNOLOGY
D. skimmed milk
E. filled milk B 96. True of Anaphylaxis:
B 77. Amount of nitrogen accumulated compared with nitrogen absorbed which indicates A. bronchospasm specific for illness
effectiveness of utilization: B. Antigens have variable routes of entry
A. Protein efficiency ratio C. Common in pediatric age group
B. Biologic value of protein D. None of the Above
C. Net protein utilization A 97. True of peripheral eosinophilia:
D. Protein daily requirement A. present in atopy
E. Protein adequacy B. specific for atopy
B 78. Amino acids in excess of needs for protein synthesis are handled in the following ways, C. absence rules out atopy
EXCEPT: D. None of the above
A. excreted as ammonia C 98. The following will differentiate urticaria from Allergic Contact Dermatitis:
B. stored as amino acids in the liver A. pruritus and lichenification
C. synthesized into energy yielding compounds B. favorable response to H-1 receptor blocker
D. may be synthesized into fats C. typical morphology of distribution
E. excreted as urea D. None of the above
C 99. True of Atopic Dermatitis:
A. Characterized by typical distribution of rash according to age group
P. GROWTH/DEVELOPMENT B. Characterized by different ages of onset
C. Characterized by typical morphology of distribution
C 79. In Tanners SMR, when the areola and papilla from secondary mound, this is: D. None of the above
A. SMR-2 B 100.True of Urticaria:
B. SMR-3 A. Characterized by typical morphology of distribution
C. SMR-4 B. Characterized by pruritus
D. SMR-5 C. Characterized by typical distribution of rash according to age group
D. None of the above

You might also like